Case Studies

Pataasin ang iyong marka sa homework at exams ngayon gamit ang Quizwiz!

Crohn's Disease

.H. is a 49 year old, male who presented to the emergency department complaining of diarrhea for the past week. He also complained of abdominal pain and cramping associated with the diarrhea and has had a fever of 101.5 ̊ F for the past three days. Patient has a history of XXXX and has episodes of exacerbation followed by episodes of remission. He already underwent a small bowel resection two years ago during a previous "flare up" of his XXX e. Diagnosis: It's clear from the patient's history and symptoms that this is an exacerbation of XXX abdominal x-ray and CT scan of D.H.'s abdomen and pelvis was ordered to re-evaluate the extent of his XXXX . CT scan showed increased inflammation. XXX e is a type of IBD that often affects the small bowel. It is a slowly progressive and unpredictable disease that has periods of remission and recurrence. Skip lesions are characteristic ofXXX and in some cases can be visualized on x-rays. Some possible complications of XXX include severe malabsorption, anemia, fistulas, bowel strictures, abscesses or cancer. Currently, there is no known cure for CXXXX.

Lyme Disease

.K., a 54 year old male presented with heat exhaustion symptoms following yard work on July 4th weekend of 95-100 degree days. Patient felt lethargic, nauseous, had chills and diarrhea. After significant rehydration with water and electrolyte drinks patient began to feel better. One week later, the patient once again had flu-like symptoms, including chills that were unbearable. Patient presented to primary care office, which stated summer flu was going around, but drew blood work and he was sent home. In the next two weeks he continued to experience odd symptoms in cyclic fashion including, muscle pain and spasms, severe lethargy, headaches, fever, vision changes, severe dizziness and loss of balance as well as groin pain. Within this three week period, the patient had days when he felt completely normal and days when he could barely get out of bed. Differential Diagnosis: The patient researched potential causes of illness online and discussed his symptoms with the primary care physician's nurse. The patient had multiple blood tests done and there was a potential that he could be experiencing a severe form of the flu, cancer, multiple sclerosis, fibromyalgia or even meningitis. Diagnosis/Prognosis: Through his research and discussion with care givers this patient thought he could possibly have XXXX He resides in Western Maryland and had been golfing the week before his symptoms began. He remembered being in tall grassy areas, but also feared that he could have been bitten by an infected tick many years before. Lab results were relatively normal other than a slight increase in his white blood cell count. XXX e is usually diagnosed based on symptoms and patient history. Blood tests specific forXXX measure antibodies in the blood but take several weeks for a positive result to appear, often people with XXXse do not have a positive blood test unless they have been suffering from symptoms for months to years. This patient's ELISA for XXX disease test was negative. The primary care physician believed that this patient was suffering from an acute episode of XXXe and was probably bitten in the previous three weeks when symptoms began.

Pneumonia

.L., a 14 year old female presents to the health center with initial complaints of headache, nausea/vomiting/diarrhea and abdominal pain for the past 3 days. Patient is unable to tolerate any oral fluids. Caregiver states general malaise and vertigo with position changes. Upon interview, patient complains of an intermittent cough. No sputum is present. Upon examination oral temperature is 104.2, all other vital signs stable. Crackles noted bibasilar upon auscultation of lungs. Pt complains of tenderness in right upper abdominal quadrant in addition to minimal abdominal cramping. No other complaints. Pt has been unable to take antipyretics related to nausea. Attempted Pedialyte at home for hydration but was unsuccessful. Differential Diagnoses: Acute appendicitis, Gastritis, Diagnosis: Patient was sent for chest x-ray. Results showed bilateral lower lobe infiltrates. Complete blood count (CBC) drawn. White count elevated at 14,000 white blood cells per microliter.

Prolactinema

.M., a 33-year-old female presents to the OB/GYN office complaining of amenorrhea and nipple discharge. Patient reports that she has been trying to get pregnant for the past 8 months so when she missed her period 5 months ago, she assumed she was pregnant; however, she reports that she has taken multiple pregnancy tests since her last menstrual period, all producing negative results. Patient reports having a normal 28-day menstrual cycle starting when she was 13-years-old with 5 day periods and mild cramping, headaches and breast soreness before each menses. Patient reports having her last regular menstrual period 5 months ago with no noted abnormalities. She reports that she has never been pregnant before and has taken oral contraceptives since she became sexually active up until 8 months ago when she and her husband began trying to get pregnant; and therefore, she is unaware of any underlying fertility issues. Patient describes nipple discharge as "white discharge that leaks from the breast randomly every day". Patient denies any pain, swelling, tenderness or redness on nipples. Patient reports a decreased libido and vaginal dryness with intercourse and denies any other sexual dysfunction. Patient denies any other health changes, including regular headaches, weight changes, fatigue, depression, visual changes and/or changes in facial bone structure. Past medical history is insignificant and family history is significant only for colon cancer in two second degree paternal relatives. Physical exam reveals normal pelvic and genital development with Tanner Stage 5 breast development and Tanner stage 6 pubic hair development. Breasts are non-tender with no redness or swelling around nipples. Small amount of white, milky discharge is noted around nipples bilaterally. Palpation and stimulation of breasts elicited additional discharge secretion. Physical exam also revealed thin, dark hair on face in male distribution pattern, which patient described to be a recent development. Remainder of physical exam is benign. Patient denies use of any medications. Differential List: presenting with galactorrhea and amenorrhea is the likely diagnosis due to history and physical exam findings. However, laboratory data and MRI imaging will be necessary for diagnosis. Other causes of be investigated before a diagnosis of can be confirmed. Renal failure, for instance, can elevate prolactin levels and, therefore, must be ruled out before diagnosis. Diagnosis: Laboratory tests reveal: negative pregnancy test; elevated prolactin (250 ng/ml); low FSH and LH levels due to suppression of GnRH; normal kidney function; normal thyroid function; normal liver function. Radiology tests: MRI with contrast of the sella turcica reveals a pituitary microadenoma (estimated size is 6 mm in diameter).

Tuberculosis

.S., a 36 year old female presents to the ED with complaints of cough times 2 weeks, and shortness of breath on exertion and fatigue. On the history you find out that the cough is non-productive. She is experiencing loss of appetite and has lost approximately 7 pounds over the last 2 weeks. During the night she experiences fevers to the point that four times over the last 2 weeks she has needed to change her sheets due to sweating. Approximately 3 weeks ago she traveled to eastern Russia. Vitals are HR: 102, RR: 22, SaO2 96% Room Air, BP: 128/78, Temperature 38C. Differential Diagnosis: Pneumonia, Bronchitis, Labs: CBC: elevated WBC of 13.1 CMP: normal Blood Cultures sent off Sputum Cultures (including AFB) PPD placed on left forearm to be read in 48-72 hours. Chest X-Ray shows a consolidation in the upper right lobe and hilar enlargement.

Meningitis

.W., a previously healthy four-week old female infant is rushed back from triage and the triage nurse states "I think she had a seizure", but the patient is without seizure activity by the time she reaches the resuscitation room. The patient is awake, but with an irritable, shrill cry. She is tachypneic, and expiratory grunting is noted. The patient is pale, mottled, and peripheral capillary refill time of 5 seconds. The patient is hypotonic with an exaggerated startle reflex, and is easily startled by even subtle movements and sounds. The patient's pupils are equal and reactive. A bulging anterior fontanel is noted. The patient is calmer when lying alone on the stretcher; when she is held by her mother or touch by anyone, she is inconsolable. The patient's heart rate is between 180-200 beats per minute, sinus tachycardia without ectopy is noted on the monitor. Her pulse oximetry reading is 100 percent on room air but supplemental oxygen is administered. Her rectal temperature is 101.2. Her mother states she only noticed that the patient exhibited lethargy, poor feeding, a few hours earlier and that she developed a fever (100.9 at home) only one hour prior to arrival to ER. Differential Diagnoses: Neonatal sepsis, , encephalitis, febrile seizure, subarachnoid hemorrhage, and brain tumor. Diagnosis: Several laboratory tests were performed, including a complete blood count, serum electrolytes and glucose, blood culture, urinalysis, and urine culture. A lumbar puncture is performed; the patient's cerebrospinal fluid is purulent, yellow in color, and cloudy. Several tubes of the CSF are collected, including a CSF culture. A diagnosis of neonatal meningitis/neonatal sepsis is made. The lab results confirm fulminant sepsis, which requires aggressive treatment. See significant results below: Complete Blood Count with Diff: WBC 1.3 K/uL, RBC 3.01 K/uL, HGB 10.1 g/dL, HCT 29.3%, MCV 87 fL, MCH 33.7 pg, RDW 16.4%, PLATELET CO 127 K/uL, MEAN PLT. VOL 8.4 fL, NEUTROPHILS 14.0%, LYMPHOCYTES 78.0%, MONOS 4.0%, Basic Metabolic Panel SODIUM: 140 mmol/L, POTASSIUM: 6.5 mmol/L, CHLORIDE: 109 mmol/L, CO2: 18 mmol/L, GLUCOSE: 81 mg/dL, BUN: 11 mg/dL, CREATININE: 0.4 mg/dL, CALCIUM: 9.1 mg/dL Blood Culture (results from 48 hrs after ED admission): Streptococcus beta-hemolytic, group B Cerebrospinal Fluid Analysis GLUCOSE: 0 mg/dL, PROTEIN: 338 mg/dL, APPEAR. cloudy AB, CSF RBC 3 /mm3, CSF WBC 77/mm3, POLYS 53 %, LYMPH 26% CSF Culture and GM Stain: Heavy growth of Streptococcus beta-hemolytic, group B Many Gram positive cocci chains on Gram stain CT Scan: No abnormal masses or midline shift. No axial fluid collections. Impression: normal examination

Meningitis

34M fevers, head ache, a stiff neck, and photosensitivity. blood cultures, urine culture, sputum culture, and a spinal tap was performed. Diagnosis: started on IV ceftriaxone and vancomycin.

Chronic Hepatitis C

35M, inebriated, complaining of right abdominal pain, fatigue, nausea and vomiting, and itching of the skin, drinking up to a bottle of vodka per day. skin and sclera were mildly jaundiced, ascites was present, 102°F, hepatic panel, ELISA, RIBA, and HCV RNA, and coagulation panel was taken. The ELISA, RIBA, and HCV RNA all came back to be positive for hepatitis C antibodies. AST/ALT ratio- 2:1, AST and ALT levels were elevated; prothrombin time was prolonged; hyperbilirubinemia, hypoalbuminemia, and hyperammonemia were all present. Cirrhosis common

ARDS

53M, fever, cough, sore throat, runny nose, body aches, headache, chills, fatigue, shortness of breath, and progressive difficulty breathing ,labored breathing with accessory muscles, crackled lung sounds, HR: 104, BP: 78/48, SaO2: 80% on 100% non-rebreather, temp: 37.9°C. Became more lethargic, produced incomprehensible words, and went into an unresponsive state, Differential list: Flu, ARDS Diagnosis: * Chest X-Ray - bilateral infiltrate * ABG - respiratory alkalosis - pH: 7.48, PaCO2: 25mmHg, PaO2: 78mmHg, HCO3: 28mEq/L * PAN cultures: Sputum: + for H1N1 * Echocardiogram - negative (r/o congestive heart failure) * Bronchoscopy * CT Scan * CBC and CMP

HIV/AIDS

57M, persistent/productive cough, sore throat, oral ulcers, weakness, fever and report of consistent weight loss. hospitalized after a car crash in 1982 received 2 units of PRBCs and has never visited a dentist. pale, diaphoretic and emaciated. Differential Diagnoses: Infection (systemic, respiratory, or GI) Tuberculosis Cancer HIV infection Diagnosis: 6 episodes of suspected undiagnosed pneumonia since 1983 and has been "sick for as long as he can remember." swollen lymph nodes, bilateral weakness of all extremities, severe cachexia, and small round, bluish-red lumps the size of peas all over the inner and anterior thighs. positive for HIV, and his CD4 count is 7 cells per microliter, well below the cut-off for AIDS (200 cells per microliter). The lumps on his thighs are determined to be Kaposi's sarcoma, a cancerous tumor of the connective tissue commonly affecting persons with weakened immune systems. A sputum culture reveals TB and a chest X-ray shows extensive damage to the lung tissue, over 75%.

Toxic Shock Syndrome

6-year old, B.J. presents to ED with sudden confusion. Her mother states approximately 1 hour ago B.J. was speaking to her when suddenly her sentences became jumbled and inappropriate. Currently B.J. has a staring gaze, responds inappropriately to commands and is not oriented to person, place, or time. She is pale with a slightly yellow cast, her skin is clammy, the palms and soles of her feet are covered in a sunburn like rash, and her conjunctiva and mouth are erythematic. Mom states that B.J. has been feeling ill for past 2 days with nausea, vomiting, diarrhea, general malaise and body aches and thought she was suffering from the flu. B.J. has no significant medical or surgical history. No recent travel. Upon further questioning mom states that she was concerned that B.J. also started having a very heavy period approximately 4 days ago and she is concerned with blood loss. B.J. proceeded to have a 45 second seizure and maintained a postictal state for approximately 2 hours. VS: Temp-103.2, BP-74/38, Pulse-110 and thready, RR 14 Differential list: Septicemia/Shock Diagnosis: The following labs were ordered: CBC, CMP, Creatine Phosphokinase, and cultures of urine, blood, vagina and CSF. CBC - WBC 22,000/mm3, platelets 89,000/ml CMP- Total Bilirubin- 3.1mg/dl, ALT 75 U/l, AST 176 U/l, BUN 41mg/dl, Creatinine 3.6 mg/dl Urine Culture - unremarkable Culture of vagina - During sterile speculum exam, 7 ml of pus and a tampon were found and removed. Site positive for Staphylococcus aureus Culture of blood - positive for Staphylococcus aureus Culture of CSF - negative Creatine Phosphokinase -300 U/L Based on the presence of Staphylococcus aureus in blood and vaginal cultures, and the other symptoms, the diagnosis was

Myasthenia Gravis

62 yr old female, K.W. with an insignificant PMHx, presents to the to the ED with complaints of progressive weakness/fatigue for 6 weeks that is worse in the afternoon, difficulty keeping eyes open, trouble with her gait, and now complains of difficulty breathing. She is intubated for airway protection and is admitted to the Neuro Surgical Intensive Care Unit. DIFFERENTIAL LIST: Guillian-Barre, botulism, LABS & TESTS: Labs: Na+ 136, K+ 3.8, WBC 10, HCT 38, HGB 13 Vital Signs: HR 73, BP 132/67, SpO2 90% on 6L NC, RR 31 Diagnostic Tests: AChR Antibody blood test - positive Nerve Conduction Study - decreased Physical Exam: Admitting Exam: K.W. is AAOx3, follows commands x4, has 3/5 strength in all extremities, and has bilateral ptosis. After 3 weeks on your unit, K.W. is now 1/5 strength in all extremities, cannot open her eyes, and now has a tracheostomy.

Mononucleosis

9M fever, chills, generalized malaise, fatigue and sore throat for the past 7 days. Pt states that he is unable to eat + pain w/swallowing related to sore throat. unable to walk far without feeling like he is going to collapse. Pt denies cough. +4 B/L swollen cervical lymph nodes. Tonsils +4 B/L with exudate. Pt had positive pharyngitis. Pt's skin is pale and diaphoretic. positive upper left-sided quadrant abdominal tenderness. HR: 103 bpm, BP: 126/80, T: 102.0 F, POx: 98%, RR: 28. Differential List: Streptococcal infection, Lyme disease, mononucleosis Diagnosis: Rapid Mono Test: Positive. Rapid Strep A Test: Positive. CMP and CBC w/differential were ordered. WBC elevated. CMP was WNL. Liver function was WNL. Diagnosis: Mononucleosis and secondary infection of Streptococcal A. Streptococcal A bacterial infection is frequently concurrent with Mononucleosis viral infection.

Diffuse Large B-Cell Lymphoma

A 28 year old male presented to his general practitioner for increasing abdominal pain over the past two months and early satiety. The patient's past medical history was notable for HIV and hepatitis B, diagnosed in 2008 after a routine blood test. A CT scan of the abdomen and pelvis showed multiple soft tissue masses and lymphadenopathy. The largest masses were in the right abdomen (4.7 x 7.9cm) and right inguinal canal (4.9 x 5.7cm). The patient was referred to a hematologist and was admitted to a hematology/oncology unit Diagnosis The patient was diagnosed with HIV-fter a biopsy of the tumor. This is an aggressive, fast growing cancer of the B-lymphocytes. A bone marrow biopsy was done to detect whether the cancer had spread to the patient's marrow; the results were negative. Additionally, the patient presented with a creatinine of 1.7. A MAG3 renal scan showed partial obstruction of his right ureter, which impaired the right renal function to 33%. The patient then had a stent placed in his right ureter to improve function.

Kawasaki's disease

A previously healthy four-year-old male, S.T., presents to his primary care physician with a fever of 104.1°F. Per his mother, the child has been febrile for one week, and his fever does not seem to go away with Tylenol® or Motrin®. Upon exam, the child has bright red, chapped lips and an abnormally red tongue. The palms of his hand are also red and peeling. Additionally, he appears irritable and lethargic, and his cervical lymph nodes are swollen. The physician suspects XXXX e, an autoimmune condition characterized by inflammation of the blood vessels, based on the presence of the palmar rash and the "strawberry tongue." Because of the risk for aneurysm in patients with XXX , he suggests that the patient be admitted to inpatient pediatric unit at the local hospital for further testing and evaluation. Differential Diagnoses measles, scarlet fever Diagnosis Once admitted to the pediatric unit, the physicians there ordered a number of labs to help diagnose S.T.'s condition. They obtained blood and urine cultures to rule out the possibility of a bacterial infection, and also tested for a slew of viral infections. All of these tests were negative. The pt's CRP and ESR were both elevated, and his CBC revealed a slightly elevated white blood count. All other labs were within normal limits. Based on the concern for XXX the patient also underwent a 12-leak EKG, which revealed normal sinus rhythm, and subsequently an echocardiogram, which was benign.

Addison's disease

A.F. a, 46 year old female patient arrived to the ER after becoming confused and incoherent to family and having a syncopal episode. Upon arrival to the ER the patient was hypotensive at 75/45, tachycardic at 123, temperature of 100.3 and RR of 20. Blood glucose was 56. Per family account patient recently had a cholecystectomy that was indicated for patient's severe abdominal pain, nausea, and vomiting. Post-surgery patient continued to have the same complaints and condition deteriorated further with symptoms of fatigue and loss of appetite becoming more severe as reported by family. Differential list: Infection/sepsis related to surgery, acute hypoglycemia, acute appendicitis, adrenal insufficiency Diagnosis: Blood cultures, CBC, and BMP were initially drawn. IV was started with 0.9%NS to maintain blood pressure and heart rate. 5% Dextrose was also administered to normalize blood glucose levels. CBC was within normal limits. BMP results showed hyperkalemia (6.0 mM) and hyponatremia (127 mM). Upon admittance to the hospital an endocrinologist ordered cortisol and ACTH levels drawn. Cortisol level was 2.6 mcg/dL. Levels less than 3.0mcg/dL are indicative of XXXX disease. ACTH levels were 60 pg/mL. Normal ACTH levels are 50pg/mL. Simultaneous low cortisol levels and elevated ACTH levels demonstrate adrenal insufficiency.

ncontrolled Diabetes Mellitus type 2

C.C., a chronically homeless 46 year old obese female arrived at the ED with complaints of severe onset of pain in her left leg and foot, chills, and appeared obtunded. Her initial vital signs were BP: 97/42, HR: 117, T: 38.2, RR 30. Assessment revealed edema, several deep ulcerations (from her left foot to tibia), and skin with a dark blue/black appearance. The wounds had a brown-red as well as purulent discharge, and a foul smelling odor. The foot and ankle also exhibited crepitus when lightly palpated for pedal pulses. Differential diagnosis: Septic Shock, gas gangrene, necrotizing fasciitis Co-morbid diagnoses: Peripheral vascular disease, chronic kidney failure, Tests: CBC, LFTs, wound and blood cultures x2, later X-ray, CT, and ABGs, and lactate levels. Notable results: WBC 17,000; H/H 7.5, 27%; BUN 68; creatinine 6.7; K+ 7.2; + Clostridium perfringens; lactate 4; pH 7.35; CO2 46; HCO3 17; blood glucose 267.

HIV

Patient Presentation 23M: diarrhea (>12 BM's per day), vomiting and anorexia. VS: T 99.8, HR 110, RR 16, POx 98% on RA. CBC, BMP, and stool cultures. elevated creatinine 2.8, Na+ 156, and decreased K+ 3.1, . Diagnosis: Admitted 4 dehydration with mild renal failure. His stool culture: positive for Shigella and he was started on Cipro®. diagnosed a year ago after testing positive for syphilis, CD4 of 202 (17%) and Viral Load (VL) of 5637

Guillain-Barré

C.F., an 18 year old male with no significant past medical history, awoke on a Saturday morning with numbness and tingling in his hands. Later that morning, he was noted to have a hoarse voice and was brought to an emergency room by his parents, only to be discharged home with no tests obtained or prescriptions given. Later that night, he awoke with shortness of breath and was brought back to the emergency room where he was intubated for airway protection. He was then transferred to another facility where he was treated for status asthmaticus and placed on Fentanyl® and propofol drips for sedation. Three days later, as the sedation was weaned, he was noted to have no movement and was not responding appropriately to stimuli; appearing paralyzed with cranial nerve palsy. He was then transferred to another facility that had a strong neurology service and could provide a higher level of care. Differential List Several different diagnoses that could be the causes of paralysis were discussed when the patient was admitted. These included botulism, myasthenia gravis, Lyme disease because the patient was noted to have a red induration on his right arm, or XXXX after it was noted he recently underwent a cycle of Augmentin® to treat an ear and sinus infection a few weeks prior. Diagnosis Several tests were performed to diagnose the problem; An MRI/MRA of the head and neck was obtained that was WNL. Lyme serologies were negative. The medical team also performed a caloric reflex test on the patient with results consistent with brain death. An EMG was obtained that showed AMSAN (Acute Motor and Sensory Axonal Neuropathy). A repeat lumbar puncture showed protein of 75 (elevated) in the CSF. These results plus consideration of presenting symptoms is what lead the neurologists to the diagnosis of XX

Lyme Disease

D, a 55 yo healthy female presents to her physician: She pulled 3 ticks off her legs after walking her dog 10 days prior to this encounter. C/C: fatigue, soreness in her joints, and rash (not the typical "bull's eye" that is present in approximately 9% of cases) at the site of the tick bite. Her pain was constant/dull and only relieved minimally with 800mg Motrin PO TID. Her pain was rated as 6-7 on a 0-10 pain scale (0= no pain, 10= worse pain ever). She denied F/C/N/V/D. · Physical exam revealed rash and joint tenderness · Pert Negatives: Subjective absence of fever · Pert Positives: H/O tick bites, fatigue, joint tenderness, rash

Lyme disease

DJ, a 28 yo healthy female marathon runner presents to her physician: DJ noticed her knee was swollen a month prior but blamed it on her marathon-training program. CC: Fatigue. Of late, she has intractable fatigue and has had to take a 2-week sabbatical from work because she "is just unable to function." By this time one month had lapsed since onset of initial s/s (swollen knee). · Physical exam was unremarkable · Pert Negatives: No rash, no history of tick bite · Pert Positives: Intractable fatigue

Diabetic Ketoacidosis (DKA) in Pediatric

E.C. is a 15 year old female who presents to the ED at 21:00 with her mother. E.C. complains of vomiting on and off over the past 2 days, a headache, and "not feeling well." She decided to come to the hospital when she started to have abdominal pain and was having trouble catching her breath. On initial assessment, E.C. is a thin girl who appears lethargic and to be having Kussmaul breathing. When the RN takes E.C.'s vital signs she notices E.C. to have a fruity odor to her breath. Her vital signs are: 36.5 degrees Celsius, HR 115, BP 90/50, RR 34, O2 saturation 94%. The RN instantly collects a finger stick blood glucose due to the fruity odor of E.C.'s breath to find a glucose level of >500 mg/dl. Other labs and cultures are collected and are notable for large urine ketones, a pH of 7.25 and HCO3 of 12. E.C. is started on IV hydration and is administered a dose of SQ Humalog®. Her glucose is checked again in 30 minutes and has dropped to 400 mg/dl. E.C. is transferred to the pediatric intensive care unit in order to be started on a continuous insulin drip and for continuous monitoring. Differential Diagnoses newly diagnosed type 1 diabetes mellitus, sepsis, dehydration, metabolic acidosis Diagnosis Labs on the unit include: Accu-chek® for glucose, CMP, CBC, ABG (or VBG if preferred), and blood cultures x2 Other tests on the unit include: continuous EKG, abdominal studies, urinalysis, urine culture, dipstick for ketones at bedside with each void, and urine pregnancy test Most important results: glucose > 400, large ketones in urine, pH 7.3, bicarbonate 15, sodium 132, potassium 4.7, no abdominal/GI complications, and pregnancy test negative

Guillain-Barre

E.K., a 35 year old female presented to a local ER with a few days of nausea and vomiting as well as general lethargy. Patient was given IV fluids and sent home with diagnosis of a GI bug. A few days later the patient woke up with tingling and weakness of lower extremities and called 911. She was admitted to the hospital on a general medicine floor for a neurology work up. Patient continued to have weakness progressing up to her torso and upper extremities and was placed on oxygen when she began to have difficulty breathing. The patient was then transferred to the ICU and was intubated due to paralysis ascending through her body. After intubation, she was unable to move any extremities other than a slight twitch of a muscle in one arm and one leg and could nod her head yes or no. Patient remained on the ventilator with full vent support and required a continuous fentanyl drip and Ativan® drip for pain and ventilator comfort. Differential Diagnosis When the patient was initially admitted to the medicine floor many ideas were discussed regarding her diagnosis. Multiple sclerosis, ALS, severe flu-like illness or XXX associated with her recent GI illness could all be possible causes for her paralysis. Diagnosis It was determined through symptom analysis and a spinal tap that showed increased protein, that the patient was suffering from XXX, an autoimmune disorder that attacks the peripheral nervous system following an acute illness. If treatment was begun immediately the patient should have a positive outcome.

Staphylococcus aureus infection

E.S. is a 25 year old female. The patient is in overall good health with no significant past medical history other than an appendectomy in 1997. She is allergic to tomatoes resulting in hives. The patient presented to the ER with complaints of itchy bug bites or pimples to the left leg. Her vital signs were: BP 104/76, HR 68, RR 14, oxygen saturation 99% on room air, and temperature 97.0 oral. E.S. denied pain. Upon assessment the patient had a 24cm x 18cm abrasion on the outside of her left calf. Above and below the abrasion were reddened areas with pustules. Differential list: Bacterial infection: Diagnosis: The patient was asked further questions about the abrasion. Approximately 10 days prior she sustained a turf burn while playing soccer on a turf field. The patient continued to play the remainder of the game. She cleaned the wound when she arrived home that night and placed Neosporin®. She continued to work out daily and play soccer. The small pustules appeared a short time later. Her boyfriend recognized the infection and encouraged her to seek help. While in the ER the nurse cultured her wound by swabbing the reddened area and placing the specimen in a sterile container. It was then sent to the lab. Results can take up to two days.

Clostridium difficile infection

E.S. is a 68 year old male admitted to the Intermediate Care Unit (IMC) status post Whipple procedure. Past medical history includes kidney stones in 2006, hyperlipidemia, and hypertension. He is a full code with no known drug allergies. The purpose of the Whipple procedure was to remove a benign tumor at the head of the pancreas. This required rearranging and rerouting the pancreas, gallbladder, small intestines, and stomach in order for the patient to digest food. The surgery was successful and the patient was admitted to the IMC for observation and started on IV antibiotics of Zosyn® and Rocephin®. On post-op day three the patient was complaining of increased abdominal discomfort. Upon exam his abdomen was non-tender and soft with active-normal bowel sounds. The patient's vitals signs were: BP 138/86, HR 106, RR 18, oxygen saturation 99% on room air, and temperature 99.9 oral. Morning labs showed an increase white blood cell count of 19,000 cells/µL. Additionally the patient had 8 liquid bowel movements in 24 hours. Differential list: Infection, electrolyte imbalance, malnutrition Diagnosis: A XXX e was collected, placed on ice, and sent to the lab for testing. A day later the patient was diagnosed with XXXand placed on contact isolation. He was given oral vancomycin and Flagyl®. A rectal trumpet was placed to prevent frequent clean-ups and to decrease infection. Due to increased fatigue and nutritional intake, a corpacks was placed and he was given Promote® tube feeding at 60cc/hr around the clock. The patient was placed on an electrolyte replacement protocol in which a BMP is drawn every morning. Based on the lab results potassium, magnesium, and phosphate can be replaced daily without additional doctor's orders.

Acute Lymphocytic Leukemia

Initial patient presentation: In June 2007, 25-year-old C.H. began to experience symptoms of fatigue and body pain. She had been studying intensely for a rigorous licensing exam and was under significant stress. During the two-week period leading up to the exam, she noticed the appearance of petechiae on her legs, arms, and stomach, and eventually her entire body. She also noticed the formation of large bruises all over her body although she had never been prone to bruising and couldn't remember any recent injuries. Her symptoms progressed to where she began to bleed incessantly from her nose and gums. She developed hemoptysis from the blood originating in her upper respiratory tract. Diagnosis: She presented to the emergency room on a Saturday night, where lab work, including a complete blood count, was performed. Her WBC count was elevated (30,000/µL) and her platelets were extremely low (9,000/µL). She immediately began receiving blood transfusions to restore her platelet levels. She was monitored closely overnight and on Sunday, she was transferred to the hospital's CCU where a bone marrow biopsy was performed. The results of the biopsy revealed ith the presence of the Philadelphia chromosome. She was later transferred to another hospital with an inpatient cancer care unit.

Diabetes Insipidus

J.M. is a 33 year-old female patient who recently relocated to the United States. J.M. had no significant medical history besides complaints of headaches which have increased in severity over the past year. Recently J.M. had stated that these headaches had become significant enough that it was affecting her quality of life and she came into the local ER for a medical evaluation. At that time J.M. was found to have a pituitary tumor which was diagnosed via a head CT scan. A craniotomy was performed and the tumor was removed successfully. J.M. was then transferred to the Surgical Intensive Care Unit for hourly neuro checks and strict recording of intake and output. J.M. arrived in the unit within one hour of completion of her surgery, and at that time was alert and oriented with urine output of 50 - 150ml per hour. However, J.M.'s urine output started to increase in the following hours with a peak urine output of 700 -1500ml per hour. The neurosurgeon on call was informed and orders for hourly urine sticks for urine specific gravity were ordered along with q 6 hour serum osmolarity and electrolytes tests. J.M. soon developed severe thirst and was consuming around 600ml of fluid by mouth hourly in addition to 200ml of normal saline intravenously per hour. During this time J.M.'s urine specific gravity decreased to below normal levels while her serum osmolarity increased to above acceptable limits. Diagnosis & Treatment: The diagnosis of X as assigned to J.M.'s condition and treatment was started to correct the fluid imbalance. XXX is a condition where the patient's kidneys are unable to conserve water, and in J.M's case this was caused by the trauma inflicted on her pituitary gland during surgery. Antidiuretic hormone (ADH) which is produced in the hypothalamus and stored and released by the posterior pituitary gland is responsible for controlling the amount of water that the kidneys conserve. In J.M's case the pituitary gland was unable to release appropriate amounts of ADH resulting in a severe imbalance in intake and output. The treatment of choice in this case was a regimen of vasopressin given subcutaneously every 6 hours based on J.M.'s lab values. Neurosurgery ordered a sliding scale of SQ vasopressin to be regulated by the lab results of J.M. J.M. responded well to the vasopressin and within several hours her intake and output equalized. According to the neurosurgeon this condition would be self-limiting for J.M. and she would be able to be taken off the vasopressin relatively soon. Since the condition was caught quickly the patient did not suffer any effects of dehydration or any severe electrolyte imbalances, two of the most common complications from this condition. J.M. was transferred out of the unit within two days and was scheduled to be discharged within the week. A follow up MRI showed the surgery to be successful in completely removing the tumor.

Parvovirus B19-Induced autoimmune response

K.B., a 26 year-old healthy young woman, went on a service trip to Peru in early May. K.B. felt fine upon her return to America for the first two weeks. Starting May 30th, K.B. slept for a minimum of 16 hours everyday. This sleeping pattern continued for approximately two more weeks. On June 1st, K.B. experienced severe pain in her left wrist. On June 7th, an x-ray of the wrist was taken. The x-ray looked fine, and the pt was given an NSAID and a splint and sent home. From June 9th -August 19th, the pt experienced the following symptoms: syncope upon waking in the morning, migratory polyarthritis, erythema and swelling of joints, pleuritis, photosensitivity, severe mouth sores, and chronic fatigue. Throughout the three months, K.B. was seen by her PCP, infectious disease doctors, and rheumatologists. Differential List A myriad of doctors throughout the summer assessed K.B. and checked her blood for the following possible diagnoses: systemic lupus erythematosus, autoimmune hepatitis, rheumatory arthritis, HIV, adenovirus, enterovirus, dengue fever, lupus-mimic induced by parvovirus, and viral-induced autoimmune response, with EBV, XXX, and Lyme disease as potential culprits. Diagnosis The current belief is that K.B. is suffering from an autoimmune response induced by XXs that she contracted while in Peru.

Syndrome of Inappropriate Antidiuretic Hormone (SIADH)

K.G., a 54 year-old male with a pack-a-day, 30-year history of smoking and small cell lung cancer (staging: extensive) presents to the oncology outpatient clinic for lab work and physical following chemotherapy two weeks earlier. Patient presents with 12 lb. weight gain since the week before, increased weakness/fatigue, and reports muscle cramps with occasional nausea, vomiting, and diarrhea. Pt is lethargic but AAOx3, with occasional headaches. No edema noted. Patient's wife reports patient has "been urinating less frequently and even accidentally peed the bed once yesterday. This is so unlike him." Patient is admitted to oncology unit for a full work-up. Differential list Differential diagnoses include: dehydration, fluid retention, cardiac disease, hepatic dysfunction, adrenal insufficiency, renal disorders, thyroid disease, and/or poorly managed chemotherapy side effects. Diagnosis Lab results/Tests: CBC, CMP, plasma osmolality, UA, urine sodium and urine osmolality, blood cultures. Significant lab results: Serum Na+= 123 mEq/L. Serum osmolality= 276 mOsm/kg. Urine osmolality= 559 mOsm/kg. Urine sodium= 24 mEq/L. (4 key diagnostic lab values: hyponatremia, decreased serum osmolality, increased urine osmolality and increased urine sodium). Electrolytes, BUN, creatinine, albumin and uric acid are also elevated. Chest X-ray to r/o pulmonary infection. CT scan was later done and revealed cerebral edema (and ruled out brain tumor/herniation). Potential causes: There is a fluid imbalance as the hormone is directing the kidney to conserve water and concentrate urine, leading to water intoxication and build-up of fluid in the intracellular space. It was determined that his SCLC tumor (similar to other malignant neoplasms) is secreting inappropriate and excessive amounts of that interfere with fluid balance. The patient's current medications are reviewed to ensure that is not medication induced (including opioids, antidepressants, non-steroidal anti-inflammatory drugs, cytotoxic chemotherapy). Other nonmalignant causes were ruled out, such as CNS disorders (infection, hemorrhage, trauma), pulmonary disorders (infection, TB, abscess) and/or pain, stress, and/or nicotine. After ruling out other causes, the diagnosis was

Syndrome of Inappropriate Antidiuretic Hormone (SIADH)

K.G., a 54 year-old male with a pack-a-day, 30-year history of smoking and small cell lung cancer (staging: extensive) presents to the oncology outpatient clinic for lab work and physical following chemotherapy two weeks earlier. Patient presents with 12 lb. weight gain since the week before, increased weakness/fatigue, and reports muscle cramps with occasional nausea, vomiting, and diarrhea. Pt is lethargic but AAOx3, with occasional headaches. No edema noted. Patient's wife reports patient has "been urinating less frequently and even accidentally peed the bed once yesterday. This is so unlike him." Patient is admitted to oncology unit for a full work-up. Differential list Differential diagnoses include: dehydration, fluid retention, cardiac disease, hepatic dysfunction, adrenal insufficiency, renal disorders, thyroid disease, and/or poorly managed chemotherapy side effects. Diagnosis Lab results/Tests: CBC, CMP, plasma osmolality, UA, urine sodium and urine osmolality, blood cultures. Significant lab results: Serum Na+= 123 mEq/L. Serum osmolality= 276 mOsm/kg. Urine osmolality= 559 mOsm/kg. Urine sodium= 24 mEq/L. (4 key diagnostic lab values: hyponatremia, decreased serum osmolality, increased urine osmolality and increased urine sodium). Electrolytes, BUN, creatinine, albumin and uric acid are also elevated. Chest X-ray to r/o pulmonary infection. CT scan was later done and revealed cerebral edema (and ruled out brain tumor/herniation). Potential causes: There is a fluid imbalance is directing the kidney to conserve water and concentrate urine, leading to water intoxication and build-up of fluid in the intracellular space. It was determined that his SCLC tumor (similar to other malignant neoplasms) is secreting inappropriate and excessive amounts of that interfere with fluid balance. The patient's current medications are reviewed to ensure that is not medication induced (including opioids, antidepressants, non-steroidal anti-inflammatory drugs, cytotoxic chemotherapy). Other nonmalignant causes were ruled out, such as CNS disorders (infection, hemorrhage, trauma), pulmonary disorders (infection, TB, abscess) and/or pain, stress, and/or nicotine.

Gestational Diabetes

Patient Presentation: 30 year old female, K.W. presents to the hospital per physician order with increased blood glucose (300 mg/dl) and glycosuria. The patient is 5'5, 180 lbs, no allergies, only home medication is Phenergan® (12.5 mg-25 mg PO PRN), no chronic illnesses. K.W. had a twin sister, no family history of diabetes or major illnesses. K.W. is currently pregnant with her first child, G1P0, at 24 weeks gestation, no complications thus far in the pregnancy. Upon admission she has no signs or symptoms of hyperglycemia, no obstetrical complaints. Pt states feeling "fine" with only complaints of occasional nausea and increased fatigue. Diagnosis: Labs (upon admission): Glucose 350 mg/dl, BP 128/84, P 84 R 20 T 98.4ºF, OGTT with a 3 hour glucose 160 mg/dl (normal <140 mg/dl) Treatment: Humalog insulin sliding scale Insulin (Regular and NPH insulin) before breakfast and dinner, which took days to find the correct dosage

MRSA Infection

K.S., a 75-year-old female presented to the orthopedic clinic for her two-week postoperative check up after her right total knee replacement. Her previous 4 day in-hospital stay had been unremarkable in which she was discharged home with physical therapy and a continuous passive motion machine. She presented in the clinic with a temperature of 39.8 degrees Celsius with chills and general malaise. She complained of shortness of breath and a low urine output x 2 days. Her VS were: BP 91/53, P 118, RR 16 and SaO2 91% on RA. Her surgical incision was hot to touch, edematous and reddened, and had copious purulent fluid draining from the lower incision site. She also had 2+ pitting, bilateral lower leg edema. She was immediately admitted as direct admit to the hospital. Diagnostic tests Upon arrival to the hospital floor labs and tests were completed including a CBC, BMP, Coags, UA, anaerobic and aerobic wound cultures and blood cultures x 2. An EKG was completed which showed NSR and the chest X-ray was grossly normal with bilateral lower lobe infiltrates. A renal US was performed to rule out renal obstruction. A CT scan of the right knee without IV contrast (due to elevated Cr) was completed revealing an abscess to the right knee. Lab results were as follows: WBC: 18 BUN: 22 Mg: 3.0 HBG: 28 Cr: 2.0 Phos: 4.9 HCT: 8.5 Na: 129 Ca: 8.2 K: 5.3 Cl: 105 UA: Grossly normal Preliminary BC results: Gram + cocci in clusters. (final analysis 2 days later +MRSA) Preliminary WC results: Gram + cocci in clusters (final analysis 2 days later +MRSA) Diagnosis Right knee postop infection with abscess; with associated Septic Shock and Acute Renal Failure.

Clostridium Difficile (C. dif)

M.M., a 72-year-old female patient is in the step-down unit. She is currently being treated for pneumonia with intravenous antibiotics and oxygen therapy but during her stay at the hospital has developed other symptoms. She is now complaining of abdominal cramping, loss of appetite and nausea. Vital signs are as follows: heart rate 85, Blood pressure 130/70, respirations 20, oral temperature 100.7˚F. The patient's abdomen is tender to the touch and hyperactive bowel sounds are auscultated in all four quadrants. Stool is watery and seedy and bowel movements are increasing in frequency, currently at six per day. In terms of differential diagnoses, this patient's fever is likely indicative of infection. This infection could be accounted for by the pneumonia which the patient is currently being treated for. Given the fever, a practitioner may order a whole slue of cultures (urine, stool, sputum, blood, etc.). However, given the nature of these specific symptoms, it seems reasonable to believe that some sort of colitis has manifested. For this reason, the practitioner orders only the most suspicious culture, stool, before proceeding to others. The nurse practitioner has a few options as far as diagnostic testing. She suspects that this patient has contracted a common nosocomial, intestinal infection known as X. She first orders a stool sample assay for XXX toxin which comes back positive for the spore. In some cases, practitioners will also perform a colon exam (flexible sigmoidoscopy) to confirm the diagnosis. Occasionally, the practitioner will order a computerized tomography (CT) to be performed. This test would show any thickening of the colon wall or the extent of the damage caused by the organism.

Diabetes Insipidus

PATIENT PRESENTATION: 51 yr old female, K.W. with a past medical history significant for only HTN, presented to the ER with SBP 276, L sided weakness, and covered in emesis. GCS is a 7. K.W. was placed on a Cardene® gtt to control BP, intubated, and a STAT CT scan of the head was performed which revealed a right basal ganglia hemorrhage with intra-ventricular extension; K.W. was transferred to the Neuro-Surgical ICU. An external ventricular drain, (EVD), was placed in order to monitor intracranial pressures (ICP), and to drain off excess fluid in the brain. The EVD is open to drain at 0cm H20 and output is bloody. Urinary output is being monitored by a Foley catheter and is adequate at approximately 100mL/hr and yellow in color. During the 1300 assessment, 18 hrs after K.W.'s initial presentation, the urinary output is approximately 500mL and has lost some of the color. During the 1400 assessment, the urinary is now 800mL and extremely dilute. DIAGNOSIS: Labs: Na+ 162, serum osmolality 315, urine specific gravity 1.001, urine osmolality 200 Vital Signs: HR: 87, BP: 155/67 (on 5mg/hr of Cardene®), SpO2: 100% on 60% FiO2, RR: 16 on vent ICP: 18 (however K.W. has had multiple periods of sustained ICP's in the 50's) Neuro Exam: K.W. has progressed to a 3T on the GCS. Pupils are 7mm and fixed bilaterally. K.W. has no cough/gag/corneal reflexes and is breathing over the ventilator by one breath. K.W. was diagnosed with XXXX elevated ICP due to hemorrhagic stroke.

Diabetic Ketoacidosis (DKA)

PATIENT PRESENTATION: L.W., a 14 y.o. previously healthy female was brought to the ED by her family. L.W. has a 2 day history of increased fatigue, thirst, frequent urination, change in breath odor, and "not feeling right". As reported by her parents, L.W. awoke this morning with nausea, vomiting and disorientation. Upon admission to the ED, L.W.'s glucose was checked and was 612 and 609 respectively. Two PIV's were placed and L.W. was started on IV Fluids. DIAGNOSIS: L.W. was diagnosed with XXX secondary to Type 1 diabetes after multiple tests were run. Glucose from finger stick was over 600. A urinalysis was run with a result of positive ketones. Labs and an arterial blood gas showed a metabolic acidosis with abnormal electrolytes (specifically potassium, phosphorus, sodium, and sodium bicarbonate).

Behçet's Disease

Patient C.O., a 27 year old male presented to an outside hospital with increased lethargy per family for the past month, right sided weakness that has progressed to the point of difficulty ambulating, and diplopia. The patient has a history of Behçet's disease that was diagnosed 3 months prior while the patient was incarcerated, as well as a seizure disorder. The patient admits to being non-compliant with his home medications, prednisone and Keppra®. At the outside hospital a CT scan was done, where they found a left thalamic and midbrain hypo-density. He was transferred to a higher level of care facility for a neurosurgery consultation. A full set of labs were sent including a CMP and blood cultures x2. 12-lead EKG and chest x-ray were obtained at the bedside. The patient is lethargic, but is alert and oriented and follows commands on all extremities. He is weak, and is urinating on himself. He is complaining of double vision and neck/headaches. He has painful open wounds in his mouth and genitalia. An MRI is ordered to further evaluate the hypo-density. Differential List It is possible that the hypo-density is a basal ganglia mass or tumor. The MRI will be able to differentiate between mass and XXXX. Due to the stiffness and neck pain we also must rule out meningitis. Other possibilities are neuro lymphoma or a demyelinating lesion. Diagnosis Once the patient was admitted, C.O. received an MRI. The radiologist reported a left thalmus and midbrain enhancement on the T2 axial flair study, with mild mass effect, mild hemorrhage and midline shift. The enhancement was consistent witXXXX . A lumbar puncture was done at the bedside, which resulted in a cloudy CSF with WBC of 249 (elevated), also consistent with XXX Rheumatology was consulted to manage XXX Two sets of blood cultures were negative.

Hodgkin's Lymphoma

Patient Presentation A 21 year old college senior, E.K., presented to her primary care doctor with a singular enlarged cervical lymph node that had become increasingly large over a three week period. Patient reports no cold/infection like symptoms prior to the increasing size. The lymph node was not tender and other than the increasing size, did not bother this senior about to graduate. The primary care doctor sent the patient for a CT scan of the neck with IV contrast. Differential Diagnosis The primary care doctor believed the swollen lymph node to simply be lymphadenopathy, cat scratch disease or a benign swollen lymph node. The doctor told the patient she was too young and healthy to have any significant issue/disease from this lymph node and told her the CT scan was inconclusive. Diagnosis The college student returned to school and within two weeks another lump had appeared on her right cervical lymph nodes as well as her clavicle region. She decided to schedule a visit with her ear, nose and throat doctor. The otolaryngologist was much more concerned than the primary care doctor had been. He sent the patient for an MRI of her neck and also performed a needle aspiration biopsy in his office. The MRI returned suggestive of a " and the needle aspiration biopsy on her lymph node was inconclusive. The next step was a lymph node biopsy of her cervical lymph nodes. The outpatient surgical procedure was scheduled and two large right cervical lymph nodes were removed and sent for pathology. A week later at a return visit for suture removal, the diagnosis was given to this young patient, The tissue had revealed Reed-Sternberg cells as well as the Epstein-Barr virus. Reed Sternberg cells are the characteristic cell of and can only be seen under a microscope. It is believed the patient also displayed Epstein-Barr cells in her lymph nodes because she had previously been diagnosed with mononucleosis, a factor in her development of this cancer. The patient was referred to an oncologist the next day to be evaluated for treatment. Blood tests were drawn and the patient was sent for a PET/CT scan to stage the cancer. The PET/CT staged the cancer at IIA; the cancer involved more than one lymph node on one side of the diaphragm. This patient had diseased lymph nodes in her neck, clavicle area, and her chest, some the size of golf balls. The "A" in her staging meant she did not suffer from night sweats, or any other ill effects of the cancer other than the swollen lymph nodes.

Multiple Sclerosis

Patient Presentation A 25 y/o female, T.F., presents to the ED complaining of severe (8/10) pain behind the eyes with weakness and pain in her left upper and lower extremities that started 2 days before. She reports no visual changes. T.F. has a past medical history of fibromyalgia, XXXX with episodes of optic neuritis. Her most recent XXX exacerbation was over a month ago, and she received 5 days of inpatient IV Solu-Medrol®. T.F. was originally scheduled to follow up with the neurologist the day after this presentation to the ED, to discuss treatment options. Differentials XXXX Considering the patient's history of XXX, the recurrence of old symptoms (optic neuritis), and the new onset of weakness, the medical team's primary presumption was that T.F. was experiencing an XXX exacerbation. Diagnosis Neuro exam showed patient AAAOx3, follows commands, appropriate & clear speech, PERLA with photophobia, extraocular muscles intact without nystagmus, positive pain with eye movement. Face symmetrical, palate and tongue midline, hearing intact. Coordination tests: no dysmetria; finger-to-nose and heel-to-shin intact. Motor: Positive for drift in left upper extremity. Strength 5/5 right upper and lower extremities, 4/5 in left extremities; normal tone. Transfers from bed-to-chair with assistance. Sensory: decreased sensation to touch and pinprick below ankle on LLE. Positive paresthesias to touch on LUE and LLE.

Cushing's Syndrome

Patient Presentation A 33-year-old female presents with secondary amenorrhea - the absence of her menses for over a year, hirsutism, significant weight gain of 15 kg within the past 2 months, acne, significant muscle weakness, recurrent headaches, hair loss, wine-colored striae on the abdomen and extremities, facial roundness, a lump over the back of her neck, and feelings of depression. Diagnosis The patient presents with symptoms indicative of The patient was admitted to further continue the endocrine evaluation. Multiple tests were ordered to confirm the preliminary diagnosis of . The results are as follows: A chem 7 panel was ordered and the glucose resulted in 335 mg/dL (high) and all other values were WNL. The patient's complete blood count resulted in a white blood cell count of 11.28 K/uL (high) and all other values were WNL. The patient's 24-hour urinary free cortisol level was 1440 mcg/24 hours (high). Her serum cortisol levels remained elevated with results of 25 mcg/dL at 2300 and 32.3 mcg/dL at 0000 with ACTH levels of 56.5 pg/mL (high) and 86.6 pg/mL (high) respectively. The following morning her cortisol levels were 29.4 mcg/dL (high) at 0730 and 34.3 mcg/dL (high) at 0800 with ACTH levels of 61.9 pg/mL (high) and 91.4 pg/mL (high), respectively. The patient then underwent a high-dose dexamethasone suppression test in which the patient's serum cortisol level went from 21.4 mcg/dL (high) and 34.3 mcg/dL (high) to 4.8 mcg/dL the next morning. These findings were consistent with and suggestive of a pituitary adenoma. A MRI of the pituitary was obtained and showed a hypo-enhancing lesion within the pituitary, concerning for pituitary adenoma.

Chlamydia

Patient Presentation A.B, an 18 year-old college student, presented to the clinic seeking contraceptive advice. She had never had a pelvic exam, has had two sex partners in the past six months, and was not using condoms or any other contraceptives. Pt states that her periods are regular, but has recently noticed some spotting between periods. Her LMP was 4 weeks ago. Pt denies vaginal discharge, dyspareunia, genital lesions, or sores. Pt's vital signs were 118/78, pulse 74, RR 18, and temperature was 37.1C. Breasts, thyroid, and abdominal exam did not show any abnormalities. The genital exam revealed an inflamed cervix that bleeds easily, with a purulent discharge coming from the cervical os. The bimanual exam was normal without cervical motion pain, uterine or tenderness of adnexa. Diagnosis Initially the pt was diagnosed with endocervicitis from the physical and history portion. Laboratory tests included: pregnancy test: negative NAAT for XXXXX: positive NAAT for Neisseria gonorrhea: non-reactive wet mount: pH 4.2, no clue cells (epithelial cells of the vagina that are covered in bacteria, sign of bacterial vaginosis) or trichomonads (parasitic microorganism that causes trichomoniasis) KOH preparation (mixes vaginal discharge with potassium hydroxide that kills everything but yeast): negative for whiff test (means there was not a strong fishy odor) HIV antibody: negative Final diagnosis

Necrotizing Fasciitis

Patient Presentation Patient S.G., a 52 year old woman presented to outside hospital ER c/o extreme weakness/lethargy, elevated temperature for past 5 days, and newly noticed vaginal drainage with foul odor. While she denied any past medical history, she had not seen an MD since her youngest child was born 12 years earlier. S.G. was "wheelchair dependent" due to decreased mobility secondary to morbid obesity. Weight on admission was 230 kg. VS were: temp 40C, BP 92/55, HR 128, RR 34, SaO2 92% on 3L NC. Relevant labs were: WBC 24, lactate 6, H+H stable at 12 & 36. ABG showed combined respiratory and metabolic acidosis: pH 7.2, PaCO2 50, PaO2 85, base excess -14. Additionally, her HbA1C was 9%. Differential Suspected sepsis R/O STI's Diagnosis Patient was transferred to our STICU after wound and blood cultures came back positive for gram + cocci and S.G. was becoming increasingly hemodynamically unstable regardless of antibiotics. It was concluded that the patient had XXXX; initial source was debated, and the most likely candidate was Staphylococcus aureus, but definitively unknown.

Breast Mastitis

Patient Presentation C.G., a 28 y/o female presents to the CNM/WHNP office complaining of throbbing breast pain, breast swelling, and redness on her left breast for 3 days along with fever, fatigue, and chills since yesterday morning. Pt HX is significant for a spontaneous vaginal delivery 3 weeks ago to a full-term 8lb 6oz female infant. Pt has been breastfeeding her baby since birth and expressed no difficulty with feeding until now. Pt temp is 100.6F with a heart rate of 95 and a BP of 122/86. Pt rates her breast pain as a 5/10 at rest, but a 8/10 when breastfeeding. Pt explains that because of the pain while breastfeeding on the left side she was mostly feeding on the right side. Pt found some relief by placing warm compresses on the left breast, but took 2 ibuprofen yesterday afternoon when the warm compresses no longer helped the pain. Pt reports cracks on the nipples of the left breast beginning 1 week ago. Differential List Plugged breast duct Acute breast abscess Inflammatory breast cancer Diagnosis Examination of both breasts showed a hot, painful, and erythematous lobule in the upper left outer quadrant of the left breast. Small amounts of purulent discharge was noted. Subscapular and brachial lymph nodes on the left side were inflamed, palpable, mobile and non-tender. The final Dx was of the left breast. A culture of the nipple discharge was taken which revealed the presence of Staphylococcus aureus. Pt is expected to feel reduced symptoms within 2-3 days with a course of antibiotics and full recovery is expected. Pt will also receive teaching regarding methods to prevent recurrence.

Sickle Cell Anemia

Patient Presentation D.C. is a five year old African American male that presents to the pediatric emergency room with chest pain. D.C. exhibits tachycardia (HR of112 bpm), tachypnea (Respiratory rate of 30 breaths/minute), shortness of breath, and an oral temperature of 100.3 degrees Fahrenheit. D.C.'s labs are as follows Hgb 8 g/dL Hct 28%, WBC 11,000/mm3. D.C. also tested negative for ANA (Antinuclear antibody). D.C. was tested for possible infection so urine and sputum cultures were done. D.C.'s labs showed positive sputum cultures for Streptococcus pneumoniae. D.C. had an abnormal chest X-ray as a result of the infection. Diagnosis D.C. was found to be in acute chest syndrome secondary to XXX. Acute chest syndrome is caused by a vaso-occlusive crisis or infection which results in sickling of red blood cells in the small blood vessels of the lungs which leads to occlusion, stasis, and anemia. Acute chest syndrome is a life threatening condition that requires rapid medical management. A vaso-occlusive crisis is when tissue ischemia occurs as a result of sickling red blood cells which leads to pain in the involved areas. D.C.'s father was a sickle cell carrier and his mother was also a carrier for sickle cell. D.C. was diagnosed with XXX before age one. D.C. underwent a splenectomy at the age of four. The splenectomy was necessary because the spleen became congested and engorged with XXXX s. This congestion and engorgement of the spleen caused sequestration crisis (or pooling of large amounts of blood) in the spleen and then a splenectomy was done. Patients who have a splenectomy are at increased risk for infection because the spleen is a secondary lymphoid organ that protects against infection. Repeated episodes of acute chest syndrome may cause restrictive lung disease and pulmonary hypertension for patients with sXXX SXXXX is one of a group of diseases known as hemoglobinopathies in which adult hemoglobin (Hgb A) is partially or completely replaced by abnormal XXX hemoglobin (Hgb S). XXX disease refers to a group of hereditary disorders which are related to the presence of Hgb S, these include XXX is the most common genetic blood disease in the U.S. XXXX is the most common form of sickle cell disease found in African Americans in the United States.

Neurocysticercosis

Patient Presentation F.A. is a 27 year-old male that recently emigrated from Honduras. He brought himself to the hospital because for the past two days he has been experiencing a severe headache which he consistently rates an 8 out of 10 on a pain scale of 0 to 10. The headache is unrelieved with any over the counter pain medications. F.A. is also complaining of severe nausea and vomiting which started earlier today and blurry vision which started about an hour ago. Differential List CNS tumor, CNS lesion, intracranial abscess Diagnosis A CT scan was obtained and several laboratory tests were done. The results are as follows: CT scan of the Cerebrum: There is an increase in ventricular size of the lateral and third ventricles which are consistent with obstructive hydrocephalus. There are three small calcified neurocysticercal cysts located in the left parietal lobe. Two hypodense, non-enhancing lesions indicative of viable larvae are visible throughout parietal lobe. Labs: Chem 7: Chloride: 109 mmol/L (elevated), Creatinine: 0.52 mg/dL (low), other values were WNL. Complete Blood Count : White Blood Cell Count: 13.42 K/uL (elevated), other values were WNL. Hepatic Panel: Alanine Aminotransferase: 46 U/L (slightly elevated), other values were WNL. Other Tests: C-Reactive Protein, High Sensitivity, Comprehensive: 0.6, LDH, Serum: 242 U/L (elevated) Diagnosis: Hydrocephalus secondary to CNS lesions which are indicative of neurocysticercosis Neurocysticercosis is a disease of the nervous system due to the infection of the tapeworm Taenia solium, which can be found in raw or undercooked meat. The presenting symptoms depend on where the cysts are located. Common presenting symptoms are seizures, headache, or symptoms of increased intracranial pressure.

Systemic Lupus Erythematosus

Patient Presentation In January 2001, C.F., a 21 year-old female, with no prior history of disease reported "not feeling right" and complained of frequent, "on-and-off" flu-like symptoms. She reported having "ambiguous" body aches despite normal blood work. In 2002-2003, C.F.'s symptoms worsened during her first year of graduate school, forcing her to withdraw at the end of the academic year. In the Fall of 2003, C.F. reported the appearance of malar rash . She complained of alopecia -"enough to clog the bath drain"- and of photosensitivity. Her joints hurt (nonerosive arthritis), especially her fingers, hands, and wrists. And she had developed a rash with raised edges (discoid rash) the size of a baseball over her left elbow. She also reported feeling pain and stiffness throughout her body. In 2004, C.F. complained of episodes of severe weakness (due to anemia) and of feeling "very inflamed and hot to the touch." Diagnosis C.F. made an appointment to see a rheumatologist in April 2003 who XXXX. He ordered lab work to test for the presence of antinuclear antibody (ANA). ANA attacks the nucleus of a person's own cells. The results came back positive with an ANA titer of 1:320. (Titers of less than or equal to 1:40 are considered negative.) Her erythrocyte sedimentation rate was also elevated which indicates the presence of inflammation although it is non-specific. Also, her C3 and C4 (complement proteins are part of the immune system and are involved in inflammatory responses) levels were low, which is typically seen in SLE. In October 2004, C.F.'s ANA was 1:640. In June 2005, the presence of a circulating XXXXX anticoagulant was detected and her PTT was prolonged (>40 seconds) indicating clotting disturbances.

Bell's Palsy

Patient Presentation: A 20-year-old male, N.R., presents at the health clinic complaining of difficulty chewing on the right side of his mouth X 1 day, with loss of motor function within two hours. Also states he cannot whistle and his face 'feels uncoordinated' on the right side. Pt reports he is recovering from the 'worst flu he has ever had' with a severe sore throat, fever, and headache, with onset about ten days ago. N.R. states he experienced significant pain behind his right ear two days ago. Diagnosis: Upon examination, N.R. is diagnosed with XXXX Grade I on the House-Brackmann scale (grading system of I-IV denoting severity and prognosis), due to sudden onset, history of URI, unilateral partial paralysis of the face, and patient's report of post-auricular pain. Neurological examination, including all remaining cranial nerves, yields no further findings of deficits. Pt is asked to check in with clinic on a daily basis to monitor progress of paralysis. Within 8 days, near total paralysis of the right side of the face is evident, now Grade III-IV on the House-Brackmann scale, including loss of taste, drooping mouth and eyelids, flattening of the naso-labial fold, inability to close the right eye with the eyeball turning upwards when pt attempts to close eye, loss of lacrimation, drooling, loss of ability to chew, smile or frown. Marked right sided facial swelling is also evident.

Botulism

Patient Presentation: A 36 year old female, J.S., presents to the ED with her boyfriend. J.S. started to feel ill 16 hours ago with nausea, vomiting and abdominal cramping. Over the last 2 hours she has progressed to having double vision, dry mouth and generalized weakness. In her history you find out that she ate stink fish (fermented fish) yesterday with family members. J.S. is AAAOx3. At triage in the ED her vitals were HR: 86, RR: 12, SaO2 92% on room air, BP: 113/68, Temp: 36.8. While in the ED J.S. experiences increasing generalized weakness and a bilateral facial droop. J.S. is now oriented times two and verbally responsive. After one hour in the ED her vitals were as follows HR: 96, RR 8, SaO2 90% on a non-rebreather mask, BP: 96/58. Differential: Food Poisoning Diagnosis: Labs: CBC & CMP: within normal limits ABG at one hour: pH: 7.31, PaCO2: 52, PaO2: 81 HCO3 27, Toxin sent out - it takes 48 hours to get results from the CDC. EKG: normal sinus rhythm

Ulcerative colitis

Patient Presentation: A twenty-one year old female presents to her primary care physician with complaints of frequent diarrhea and cramping. Her last several bouts have included small amounts of blood, and she grew concerned enough to make an appointment. She is 5'5'', weighs 120 pounds, and is also complaining of low energy. Her blood pressure is 105/70, heart rate 85, temperature 37.9, respiratory rate 18. She states she has had no recent travel and has not been taking any medications. Differential List: Crohn's disease, , bacterial or viral infection Diagnosis: A CBC was drawn to evaluate for anemia due to the patient's complaints of lethargy and bloody stools. Her HGB was a low normal (13). Stool cultures were taken to evaluate for an infection vs inflammatory process. White blood cells were discovered in the stool culture, and a colonoscopy was scheduled. The colonoscopy revealed continuous inflammation and reddened mucosa of the recto-sigmoid colon, and biopsies were taken. Biopsies revealed inflammation of the crypts and distortion of the crypt architecture. A diagnosis of was made.

Myasthenia Gravis

Patient Presentation: A.F., a 55 year old male presents to the ER with multiple complaints, including ptosis, dysarthria, shortness of breath, and muscle weakness of his lower extremities. BP is 149/88, HR is 92, SaO2 is 93% on room air, RR is 19, and temperature is 99.3. The ptosis has been a recurrent problem for the last two years and has not been resolved after two visits to the optometrist. Within the past several months he reports increased muscle weakness and difficulty chewing and swallowing properly. Last month he was treated in the ER for aspiration pneumonia and discharged. Patient reports that he has recently been under a lot of stress. Differential list: Stroke, multiple sclerosis, brain tumor, pneumonia, Diagnosis: In the ER a chest x-ray was performed that revealed bilateral infiltrates. CBC showed an elevated WBC of 13,000. BMP revealed mild hyponatremia with a sodium level of 130. All other electrolytes were within normal limits. CT scan does not reveal infarct or hemorrhage. MRI is ordered which is also negative for infarct or hemorrhage. EKG is normal sinus rhythm. 500 mg of IV Levaquin® is started in the ER to treat the pneumonia along with a bolus of 0.9% Normal Saline. Patient is admitted from ER for pneumonia and a neurology referral is placed. Neurology sees the patient after he is admitted to the hospital and orders an AChR antibody test. Elevated serum antibodies to the acetylcholine receptors confirm a diagnosis of

Acinetobacter Pneumonia

Patient Presentation: A.G. , a 40 year old Afghan National Army male s/p IED (improvised explosive device) blast to face, chest and limbs, was being treated in a combat ICU for devastating facial and extremity injuries. Pt has been mechanically ventilated via tracheostomy since his admission due to the extent of his face wounds. A.G. is becoming increasingly febrile, difficult to wean from the ventilator and has worsening x-ray imaging studies. VITAL SIGNS: Heart Rate: 110, Blood Pressure: 92/64, SpO2: 88, Axillary Temperature: 39.0 CBC: WBC: 22, RBC: 5, HCT: 8, HGB:26, PLT: 200 ABG: pH: 7.19, PaCO2: 65, PaO2: 55, HCO3: 28 DIFFERENTIAL DIAGNOSES: Klebsiella pneumonia, MRSA pneumonia, DIAGNOSIS: Chest x-ray was performed and revealed large areas of consolidation in both lungs indicative of pneumonia. A.G. underwent multiple bedside bronchoscopies to take sputum samples and to wash out the copious amounts of secretions that he could not clear on his own. A bronchoscopy is a procedure in which the doctor uses an endoscope to look at the patient's airway, starting with the mouth and down to the lower lobes of each lung. The sputum results came back positive for multi-drug resistant XXXX XXXis a bacterium common to soil around the world, which in the past has not been a worrisome pathogen. However, in recent years, this bacterium has been causing problems in the war wounded and particularly those with IED blast injuries due to the amount of soil and dirt becoming engrained in the wounds.

Pituitary Tumor

Patient Presentation: D.A., a 46 year old female presented to an OSH complaining of recent gait ataxia, blurry vision, bilateral temporal hemianopsia, dizziness, and intermittent headache. She has a known history of a "brain tumor" and HTN. She did not complain of N/V, numbness or tingling in extremities, recent weight loss/gain, or history of falls. Differential List: (if she had not known of brain tumor already) Stroke Diagnosis: After performing a head CT without contrast at the OSH, a XXX r was seen and the patient was sent to the Neuro ICU at a local hospital for further workup and treatment. Here, a MRI of the brain with and without contrast was done. A large mildly hyper dense mass in the sella turcica and suprasellar region measuring 6.2 x 5.3 cm was found. No hydrocephalus or acute hemorrhage was found. In the MRI, the optic chiasm was not visualized, most likely being covered or displaced by the mass. Based on the location of the tumor visualized on the CT and MRI, the tumor is most likely to be a XXX. A CBC and Chem-10 were all WNL.

Colorectal Cancer

Patient Presentation: D.H. is a 54 year old, female who initially presented to her primary care physician with complaints of abdominal pain, decreased appetite and weight loss over the past 3 months. Additionally, the patient has experienced some changes in her bowel pattern and recently started having frequent episodes of blood in the stool. D.H.'s hemoglobin and hematocrit were both low at 10.5 gm/dL and 32.9%, respectively. Her past medical history includes hypertension and type 2 diabetes. Differential List: Diverticulosis/diverticulitis; inflammatory bowel disease (e.g. Crohn's disease or ulcerative colitis); Diagnosis/Prognosis: A colonoscopy was performed and tissue samples of polyps were obtained and sent for biopsy. Biopsy findings confirmed adenocarcinoma of the ascending colon and it was determined that D.H. would need to undergo surgery for resection of the tumor. s the third most common cancer in both men and women and incidence increases with age. Since this trend is observed, screening is recommended in adults over age 50. screening involves getting a colonoscopy completed every 10 years. However, adults who have a family history of CRC should begin this screening at a younger age and should have a colonoscopy completed more frequently than every 10 years. Surgery is the primary method of controlling CRC, and depending on the staging of the tumor, chemotherapy may or may not be recommended.

Leptospirosis

Patient Presentation: D.S., a 25 year old male, presented to the ER with chills, muscle pain mostly in the calves and lower back, dry cough, headache with neck pain, nausea, vomiting, conjunctival redness, and a blotchy skin rash. He thought he was just feeling under the weather after his recent trip to Hawaii, which he returned from one week ago. However, the onset of nausea, vomiting, and the appearance of the skin rash prompted him to go to the ER. His vital signs taken in the ER were: BP 115/82, HR 90, RR 16, Temp 39.2, SpO2 99%. He reports that he recently went to Hawaii where he swam in water at the base of a waterfall at the top of a mountain. He has no significant past medical history. D.S. was admitted to the hospital for further work up. Differential List: Based on the clinical presentation, these symptoms could possibly be caused by bacterial, viral, or fungal infection. The infection could be linked to diseases involving hemorrhagic fever or meningitis. Initially these symptoms could have been attributed to influenza, but the appearance of the blotchy skin rash caused that to be ruled out. Diagnosis: Upon admission, labs were drawn including a CBC, BMP, LFT, ESR, ELISA, blood cultures x2, and a UA/UC was collected. An EKG was done as well as a LP and a chest x-ray. Lab results showed increased WBCs, decreased platelets, increased BUN, increased creatinine, decreased K+, increased liver enzymes, increased ESR, and +ELISA. The EKG showed sinus arrhythmia. The chest x-ray was normal. The blood, urine, and CSF were positive for XXX bacteria. Based on this finding along with the fact that D.S. recently spent time in the water in Hawaii where this infection is prevalent, the microscopic agglutination test (MAT) was ordered. This test was positive, which is definitive of the diagnosis of XX

Turner Syndrome

Patient Presentation: H.M., a 16-year-old female presents to the OB/GYN office complaining of primary amenorrhea. H.M. reports that she has never experienced menstruation or vaginal spotting. Additionally, patient reports that her mother and sister experienced menarche at age of 13, resulting in normal reproductive and sexual development thereafter. Past medical history is significant for a heart murmur discovered in early childhood, delayed growth in development beginning at the age of four and chronic middle ear infections. Patient's medical records also indicate treatment for four UTIs over the course of the past year. Patient denies recent exposure to radiation, viral infection, chemotherapy treatment and/or surgical removal of an ovary. Family history is negative for heredity diseases. Physical exam is significant for delayed development of secondary sexual characteristics with Tanner stage 2 breast development and Tanner stage 1 pubic hair development. Patient's chest is broad; nipples are widely spaced with minimal breast bud elevation. Patient's BMI is 21, indicating a healthy weight. Short stature is noted with patient's height measuring at 4'10". Patient's first-degree female relatives have an average height of 5'7". Patient also displays disproportionately short finger and toenails. Subtle webbing of the neck with low posterior neckline is noted. Differential List: This case of amenorrhea may also be explained by anatomic abnormalities, such as congenital absence of uterus, ovaries, or vagina, or hormonal abnormalities, such as hypothyroidism, hypothalamic dysfunction, or pituitary dysfunction. Diagnosis: External and internal vaginal/pelvic exams are grossly normal; all reproductive organs are present and in correct anatomical position. External exam revealed delayed pubic hair development as previously noted. Internal exam also ruled out outlet problems, such as imperforate hymen , stenotic cervix and/or transverse vaginal septum. Pelvic ultrasound is normal, confirming normal anatomical size and placement of uterus and ovaries. Laboratory tests: CBC normal; Serum TSH normal; FSH and LH levels HIGH (levels > 50mIU/mL) Karyotype: confirms diagnosis of Turner Syndrome Based on the karyotype, the diagnosis

Diabetes

Patient Presentation: J.A., a 27-year-old female patient presented with fatigue to the local ED. Upon history she reported weight loss over the last four months with exercise but is "getting thinner" to where it is concern for her family. She reports drinking at least a liter of water daily and has frequent urination every 2-3 hours. The patient has a past medical history of gestational diabetes that was controlled by diet and resolved after birth. Denies nausea and vomiting. Differential Diagnosis: hyperthyroidism, anorexia Diagnosis: Several tests were preformed to ensure that the patient was not in immediate danger. A CMP was order along with a TSH and a CBC to check for underlying imbalances that could cause symptoms. Her thyroid was not palpable upon examination and there we no cervical lymph nodes present to suggest an underlying infection. Urinalysis was sent to laboratory as well. Patient was also started on NS 500 ml bolus @ 150ml/hr to begin to offset the signs of fatigue. Labs results as follows: H/H: 12.1/37 RBC: 4.6 MCV: 90 MCH: 28 Platelet: 250,000 Calcium: 9 Chloride: 96 Magnesium: 1.6 Phosphate: 2.6 Potassium: 3.7 Sodium: 137 TSH: 0.75 Glucose: 463 UA wnl

Graves' disease

Patient Presentation: J.R., a 32 y/o female presents with complaints of anxiety accompanied by a "racing heart." She reports a recent job loss and attributes presenting symptoms with this event. A thorough history revealed concerns of weight loss, and missed period. Physical examination reveals: HR 105 with regular rhythm, B.P. 132/90, presence of fine bilateral hand tremor, presence of perspiration, skin is warm to touch, and minimal non-tender enlargement of thyroid gland is detectable upon palpation. Differential List: Anxiety, must rule out Pregnancy Diagnosis: TSH assay 0.03 mU/L - Normal Values (0.5-5.0 mU/L) Serum Total T4 12.5 mcg/dl - Normal Values (4.6-11.2mcg/dl), T3 205 ng/dl - Normal Values (75-195ng/dl) 24-hour radioiodine uptake and scan reveals a moderate to high degree of radioiodine uptake. Normal to high levels is consistent with In patients with contraindications for radioiodine uptake (i.e. Nursing Mothers) laboratory tests for presence of thyroid stimulating immunoglobulin and ultrasound assessment of thyroid blood flow may also be useful. hCG level undetectable - (detectable levels of hCG are a positive for pregnancy) The high T3/T4 and low TSH indicate

DiGeorge Syndrome

Patient Presentation: K.S. is an 18 month old male who presented to the pediatric clinic for a well-child visit. His mother voiced concern that he was significantly delayed in achieving both verbal and motor developmental milestones. K.S. had only recently begun walking unassisted. He was not yet pointing to indicate wants or waving. "Mama" and "Dada" were the only recognizable words he used although he did babble. K.S.'s mother also reported concern regarding his height and weight, both of which were below the 5th percentile on the CDC growth curve. Past medical history was significant for low birth weight, presence at birth of a small VSD which spontaneously resolved by 4 months of age, penile chordee without hypospadias which was surgically corrected at 11 months and frequent respiratory infections and high fevers as an infant. Upon physical exam, some mild dysmorphic features were noted including micrognathia, fused lower lateral and central incisors, a left preauricular skin tag and bilateral fifth finger clinodactyly. The child appeared small and thin but well cared for, with no signs of neglect or abuse. K.S.'s mother reported that he had a good appetite and ate a variety of foods from all food groups. The patient's mother was 5'6" and father was 6'4". The patient's family history was negative for genetic disorders. Differential List: Cystic Fibrosis - Although less likely than a chromosomal abnormality, CF could account for the patient's small size despite adequate diet and frequent respiratory infections in infancy. Additionally, neither parent had been screened for CF carrier status. Failure to thrive of unknown etiology Diagnosis: Laboratory tests: CBC-WNL, CMP-WNL, Sweat electrolytes test- 25mEq/L (normal finding, not indicative of cystic fibrosis) Normal XY karyotype Array based comparative genomic hybridization (aCGH) confirmed at least 970kb deletion at 22q11.2. This is a test that is used to detect changes in a chromosome such as deletions or duplications of material. It is used when physical findings indicate likely chromosomal abnormalities. Radiographic studies: Renal ultrasound showed malrotation of left kidney thought to be a variant of normal Hearing and vision tests: normal XXX is one of a number of disorders including velocardiofacial syndrome, and conotruncal anomaly face syndrome caused by a partial deletion of the long arm of the 22nd chromosome (22q11.2). Because the phenotypes associated with these syndromes can vary widely, they were once thought to be distinct disorders. Following the discovery that all are caused by the same genetic error; a movement began to reclassify all of the known syndromes associates with the 22q deletion as 22q11.2 deletion syndrome. Although there are a large number of abnormalities linked to the 22q11.2 deletion, some characteristic findings include: immune system deficits (often only seen in infancy), hypocalcemia (sometimes resulting in seizures), short stature, skeletal abnormalities, renal abnormalities, heart defects, cleft palate, developmental delays, learning disorders and ADHD. Individuals with 22q11.2 deletions are also at significantly higher risk for developing schizophrenia as adults than those without the deletion.

Thyroid Storm

Patient Presentation: L.B., a 28 year old female presented to the Emergency Department's triage room on a hot summer evening stating: "My heart is racing and I am burning up. I feel so nauseous." The patient was diaphoretic and seemingly agitated from the triage questions. Her vital signs were: T-38.7, P-124, BP-169/92, RR-24, SaO2 - 98%, and pain 7/10 in the abdomen. Upon further triage, it was discovered that the patient was diagnosed 1 week ago with pneumonia at her PMD office and that she had suddenly lost her job 2 days ago. She reported recent weight loss along with nausea, vomiting, and diarrhea for the last few hours. Differential List: The triage nurse immediately thought of differential diagnoses in order to assign an appropriate triage level and ED room. Differentials include: anxiety attack, sepsis, drug toxicity, hypertensive encephalopathy, thyroid storm, supra-ventricular tachycardia, or meningitis. Because of the acute nature of the patient's symptoms and her unstable vital signs, the patient was made a level 2 and immediately brought to a room with a cardiac monitor. Diagnosis: In order to diagnose the patient, the MD ordered an EKG (sinus tachycardia), a chest x-ray (marked improvement from her recent pneumonia but negative for any significant findings), a urinalysis and urine toxins screen (both negative), a spinal tap with CSF analysis (negative for WBC and normal glucose levels), along with BMP (normal), CBC (mild leukocytosis), LFT (all elevated), blood cultures x2 (later found to be negative), T3, T4, and TSH (elevated T3, elevated T4 and free T4, and low TSH). The patient was concluded to have XXXX and was specifically inXXX The longer a patient goes untreated, the greater likelihood of irreversible progression to death.

Crohn's Disease

Patient Presentation: L.Y., a 39 year old male, presents to the ER with c/o severe, diffuse, intermittent abdominal pain for several months, worsening over the past week and now unbearable. The patient reports that he previously made an appointment with a gastroenterologist for the following week, but was now unable to tolerate the pain any longer. The patient rates the pain as a 10/10, sharp and cramping in nature. The patient reports being in good health with no past medical history and no other complaints. The patient denies other GI symptoms such as anorexia, nausea, vomiting, diarrhea, and constipation. Initial Vital Signs: BP 110/70, HR 74, RR 16, O2 Sat 100% on RA, Temp. 98.4. Differential Diagnosis: Gastritis, appendicitis, pancreatitis, IBD, Infectious Colitis, IBS, Bowel Obstruction, diverticulitis Diagnosis: Upon arrival to the ER, labs were obtained (CBC, CMP, amylase, lipase, ESR, and UA), an 18 gauge IV was placed, and a STAT CT scan of the abdomen/pelvis with PO & IV contrast was ordered. The results of the patient's UA, BMP, amylase, and lipase were all WNL. Abnormal lab values included: an elevated WBC count (14.7), an elevated platelet count (618), and an elevated ESR rate (64).

Diabetic Ketoacidosis (DKA)

Patient Presentation: M.B., a 28 y/o female presents to the ED with general weakness and vomiting. Her PMH includes IDDM. ROS is positive for polyuria, and polydipsia. Pt also reports dysuria that started 3 days prior. Pt denies SOB or cough, chest pain, abdominal pain, bloating, change in bowel habits; and reports taking her insulin regularly. M.B. also denies alcohol and drug use. VS: T 101.2, BP 120/80, HR 120, RR 24, SpO2 98% RA Neuro: AAOx3, but drowsy Lungs: CTAB. No wheezes or crackles. Heart: S1, S2, Tachycardic but with Regular Rhythm. No murmur, rubs, or clicks. Abdomen: soft, non-tender, non-distended. BS(bowel sounds) present. Differential List: HHS (Hyperglycemic Hyperosmolar State) Diagnosis: Tests: EKG was normal; chest x-ray was clear without infiltrate Labs: Urinalysis was positive for ketones, leukocyte esterase, nitrites, and WBCs. Na+: 149 mEq/L; K+: 3.2 mEq/L; HCO3-: 16 mEq/L; pH: 7.2; Anion gap: 18; plasma glucose: 455 mg/dL; troponin: <0.04; urine drug screen: normal; etoh: 0.0 Based on the elevated blood glucose and the high anion gap acidosis, th.

Hyperosmolar Hyperglycemic Nonketotic Syndrome (HHNKS)

Patient Presentation: N.F., a 72 year old male was found unresponsive in his home by his daughter. She had not seen him for three days and went to check on him; finding him unresponsive, she called 911. When EMS arrived, they found him to have a GCS 3, so they bag/mask ventilated him, started IV hydration, and transported him to the ED. Enroute they checked a blood glucose level, but their glucometer gave a reading of ">400". N.F. was intubated in the ED for airway protection, chemistries were drawn, and again a fingerstick blood glucose read >400. N.F.'s daughter told the ED staff that he had a past medical history of hypertension, type 2 diabetes mellitus, and alcoholism. Differential List: Diabetic ketoacidosis, Diagnosis: Labs: Na: 133, K: 4.6, Cl: 97, Glucose: 2250, CO2: 22, BUN: 40, creatinine: 2.1, serum osmolarity: 356 Urinalysis: No ketones EKG: Sinus Tachycardia, HR 132 ABG: pH: 7.41, PaCO2: 34, PaO2: 85, HCO3: 23 Vital Signs: HR: 132, BP: 87/45, Temp: 37.5, RR: 15 (on vent), SpO2: 95% on 100% FiO2 Radiology: Head CT negative Given the reasonably normal ABG and elevated blood glucose and serum osmolarity, the diagnosis was coma secondary

Nodular Hyperthyroidism

Patient Presentation: S.H., a 17-year-old female presents to her physician's office complaining of recently having found a small, palpable lump in her anterior neck. The patient also states for the past few months she has experienced feelings of her heart racing, increased sensitivity to heat, heavy sweating, and high anxiety levels that have interfered with her daily life. S.H. reports family history of paternal aunt with Graves Disease and a 23 year old female and 24 year old male siblings each with multiple benign thyroid nodules. Differential List: Graves' Disease, follicular thyroid carcinoma, hyperfunctioning thyroid nodule Diagnosis: After ultrasound confirmed the presence of a single mass along the thyroid gland, patient underwent fine needle aspiration biopsy to determine whether the mass was benign or showed changes consistent with malignancy. Results determined the mass was benign and the patient is diagnosed with a thyroid nodule. Laboratory tests drawn show T3, T4 are elevated and TSH is low (T4 total thyroxine: 14mcg/dL, T3 total: 250 ng/dL, TSH: 0.19mIU/L) S.H. also had an Iodine Uptake Scan performed. The patient was given I-123 and her thyroid uptake of the iodine was elevated. Her thyroid scan showed the nodule had a high concentration of the iodine uptake, signaling that her nodule was likely responsible for her elevated T3 and T4 levels.

Multiple Myeloma

Patient Presentation: S.K., a 30 year old female, presented to ED with chief complaint of persistent back and left sided rib pain, unrelieved with OTC medications and rest. Pt states duration of pain at 3 months, but becoming increasingly worse within last week; now rating pain 8 of 10. On physical examination, pt was tender to palpation in left upper chest, and had tenderness on the left scapular region. Pt was febrile to 38.3 and hypertensive at 168/86; all other VS (HR 82, RR 16, O2 98% RA) were WNL. Labs drawn were remarkable for anemia with Hgb 8.3 and Hct 24.3. Pt underwent a bone scan which showed a plasmocytoma in the eighth left rib and a small lytic region on the medial right ilium. Pt. was admitted for further evaluation. Differential List: Osteoblastoma, B cell non-Hodgkin's lymphoma, Waldenstrom macroglobulinemia, Diagnosis: A CT scan with IV contrast of thorax, abdomen, and pelvis was performed and again indicated a left eighth rib lesion and vertebral body lesions at T4 and T6. Pt's free Kappa Light chain urine level was 131 mg/L (elevated and indicative of plasma cell neoplasms), a 24 hour urine collection showed a protein level of 360 mg (elevated), serum IgG was 4410 mg/dl (normal range 620-1400 mg/dl), and protein electrophoresis of blood showed gamma globulin level of 2.8g/dl and total protein of 9.6g/dl (both elevated). A bone marrow biopsy showed 20% atypical plasma cells and a tissue biopsy of the eighth rib showed plasma cell neoplasms

Sickle Cell Anemia

Patient Presentation: S.K., a 48 year old African American women, presents to the ED with 4 days of intractable pain, uncontrolled by her home pain regimen (10mg oxycodone PO q2hr prn). The pt rates the pain a "10 of 10", describing it as sharp and located in elbow and knee joints and abdomen. In addition, pt notes low grade fever for past 2 days. Pt's current pain is consistent with previous episodes of XXX crisis which have been increasing in frequency; approximately one per month. Of note, her most recent hospitalization (9 weeks prior) required a blood exchange transfusion. Physical exam is significant for BP 130/60, pulse 98, RR 18, SaO2 94% on RA, temp 38.1. Heart: rate is regular, no murmurs. Lungs: Clear bilaterally, no wheezing or crackles. Abdomen: soft, tender, + bowel sounds. No edema, swelling, warmth or erythema. Labs upon arrival are significant for Hgb 8.2, Hct 23.5%, reticulocyte count 12.4%, WBC 11.4. Differential List: Septic Arthritis (If pt. had not already tested + for presence of homozygous HbS, then additional differentials would include testing for other chronic anemia's that involve abnormal Hgb such as Thalassemia) Diagnosis: Based on patients previous similarity to XXXX elevated reticulocyte count, and low Hgb and Hct, pt was diagnosed with sickle cell pain crisis. Bone marrow transplant offers the only potential cure for sickle cell disease. Due to difficulty finding donors and serious risks associated with this, the treatment for sickle cell disease is symptom management. For pain crisis, the goal is to treat pts to where they can manage their pain at home.

Cystic Fibrosis

Patient Presentation: The mother of a 2-year old Caucasian girl brings her daughter (HEU) to the pediatrician's office. The mother states that HEU has had frequent watery, foul smelling and sometimes greasy stools on and off for the past 3 months. At other times HEU seems to be constipated. HEU is in the 50th percentile for height and 25th percentile for weight. She is noticeably small for her age. The mother also reports that after HEU's last cold, the cough did not subside for several weeks and each cough was producing thick, grayish mucus. Her medical record shows 3 hospitalizations in the past 2 years from respiratory tract infections, all of which were treated with antibiotics. Today, HEU appears to be in good health with no apparent cough, but she looks malnourished and pale. Differential List: Failure to thrive Gastrointestinal infection unrelated to past respiratory infections Hirschsprung's Disease Intussusception Diagnosis: Abdominal ultrasound showed no obstructions or mechanical blockages Stool culture revealed normal bacterial growth, without viral infection Stool sample was positive for fat and vitamins A, E and D Chloride sweat test on 150mg of sweat revealed 111mEq/L of chloride (high) Gene testing determined that HEU was homozygous for mutations on the CFTR gene Upon inquiry, HEU's mother responds that HEU's skin tastes salty when she kisses her. With the positive diagnosis of (an autosomal recessive disorder), HEU and her family prepare for a life of symptom management. There is no cure for though diligent treatments allow an affected individual lead a relatively normal life.

Sickle cell crisis

Patient Presentation: A 3-year old child, K.S. presented to the ER with acute onset chest pain, coughing, difficulty breathing, and fever. She also complains of severe generalized pain mostly in her chest and abdomen that is increasingly worsening as well as debilitating. The patient's mother says she has had flu like symptoms for 2 days with a runny nose, congestion, fatigue, and decreased appetite. The mother states she has been pushing fluids but has not been too successful. Due to routine newborn screening tests, K.S. has had a XXXX disease since birth and therefore is up to date with her current immunizations; however she has had multiple hospital admissions due to anemia in the past 3 years. Her vital signs are RR 42, T 38.5 Celsius, BP 92/52, P 126, SaO2 90% RA. She is immediately placed on 4 L O2 per NC and baseline Labs, Chest X-Ray, and EKG were completed. IV NS bolus of 500 ml was given along with maintenance fluid at 50ml/hr. IV morphine was given to control her pain. EKG: Sinus Tachycardia Chest X-Ray: Bilateral lower lobe infiltrate Labs: WBC: 18,000, Hbg 7, Hct 22, Reticulocyte 0.2%, RBC 2.8 BMP: Na+ 136, K+4.0, Mg++ 1.5, Phos- 2.9, BUN 18, Cr 1.0 LFT: ALT 37, AST 46, Direct Bil 0.5, Total Bil 1.1 Diagnosis: XXX XXX is an autosomal recessive inherited disorder. Erythrocytes in sickle cell anemia contain abnormal hemoglobin that affects the beta-chain producing hemoglobin S or HbS. Valine (amino acid) takes the place of the normally appearing glutamic acid in beta-chains. Replacement of glutamic acid with valine causes the polymerization of HbS components to cohere forming long and insoluble particles. This distorts the RBC, which then assumes the inflexibility crescent or sickle shapes that become sharp and spiky when the RBC's are deoxygenated. XXX refers to episodes of acute and severe sickling that blocks the circulation posing a threat of excessive organ damage and severe pain. Sickle Cell Crisis may occur in any part of the body and may be brought on by cold or dehydration. Acute chest syndrome can be caused by a lung infection or by sickle cells blocking blood vessels in the lungs.

Systemic Allergic Reaction

Patient Presentation: K.S., a 20 yo female presents to the health clinic for weekly routine allergy injection (trees, grass serum). Peakflow: 370. Lungs clear B/L to auscultation. No SOB. T: 98.4 F. K.S. denies feeling sick today. K.S. states she took antihistamine 2 hrs prior. K.S. denies reaction/issues with prior injection (0.45ml) last week after leaving health clinic. K.S. given 0.5ml (maintence dosage) of allergy serum in left mid-upper arm. K.S. monitored for 30min after injection for any reaction. + 2cm wheal. + Erythema. No edema. No difficulty breathing. K.S. okay to leave health clinic. One hour post-allergy injection, K.S. returns to health clinic complaining of left arm, facial and lower lip edema, uticaria and erythema. K.S. denies SOB or dyspnea. K.S. denies dysphagia. K.S. denies nausea or vomitting. Upon examination, lungs clear B/L to asculatation. No wheezing, no crackles. Vital signs: HR: 110 bpm, BP: 94/60, T: 98.5 F, POx: 95%., RR: 30. Diagnosis:

Diffuse Large B-Cell Lymphoma (DLBCL)

Patient presentation A 42 year-old female (K.G.) presents to emergency department with influenza-like symptoms, swollen, non-painful lymph nodes in neck, armpits, and groin. Pt complains of fatigue, a "nagging cough," and back and abdominal pain with occasional diarrhea (sometimes with blood in it). Pt has also been having persistent night sweats, fevers and loss of weight greater than 10%. Pt has a history of colorectal cancer (treated and in remission), diabetes, and a significant alcohol/drug abuse history. Differential list Differential diagnoses include: infection (viral, bacterial, parasitic), lymphangioma, drug toxicity, Hodgkin's vs. non-Hodgkin's lymphoma. Diagnosis Lab tests: CBC with diff, peripheral blood smear (assessing for cytopenia and lymphoma cells), CMP, LFT (elevated), LDH (=634, elevated), immunoglobulin levels (elevated), Hepatitis panel, HIV (reactive). Diagnostic procedures: CT scan, PET scan; Lymph node biopsy; Bone marrow to assess involvement; LP/MRI of brain if possibility of CNS involvement. Diagnosis: (Stage IV, disease on both sides of the body and above and below the diaphragm, also GI/liver lymph node involvement) and HIV (common to have lymphoma secondary to progressed HIV diagnosis).

Gestational Diabetes

Patient presentation C.G., a 32 y/o female pregnant with her second baby (gravida 2 para 1) presents to her CNM for her 24 week appointment. Pt's height is 5ft 7in, weight is 180lbs, pulse 84, temp 97.3, and BP is 126/82. C.G.'s hx reveals a normal spontaneous vaginal delivery in May 2009 at 42 weeks gestation to a 9lb 4oz female infant. Pt reports increased hunger and craving of sugary substances even after eating, as well as increased lethargy and fatigue. The results of the Pt's OGTT were 182mg/dL at one-hour, 158mg/dL at two-hour, and 143mg/dL after three-hours. Pt did not report any increase in urination or thirst, or any weight loss. Pt denies hx of DM or FHx of DM and reports recent travel to France at which point she stopped her exercise routine and was eating lots of highly fatty and sugary foods. Diagnosis Tests: Next day overnight fasting blood glucose followed by a second 50g oral glucose load to confirm the diagnosis of GDM. Results: Fasting blood glucose taken the following AM revealed a finger stick blood glucose (FSBG) of 105mg/dL. The results of the second oral glucose tolerance test (OGTT) were one-hour 180mg/dL, two-hour 157 mg/dL, and three-hour 142mg/dL which confirmed the dx of GDM.

Non-Hodgkin's Lymphoma

Patient presentation: J.M., a 57 year-old woman with a 20 plus history of myasthenia gravis presented to the ER with increased fatigue, severe muscle weakness, and a general feeling of "sickness". The patient denied any other significant history, but was on several medications around the clock to control the symptoms of her disease. According to the patient her symptoms had been getting progressively worse for several weeks, but the patient was waiting for her regular doctor visit to be checked, as she assumed it could be attributed to the myasthenia gravis. Upon arrival to the ER the patient's blood pressure was dangerously low, with the systolic number sustaining in the 70's and 80's. Two fluid boluses of normal saline (500ml each) were administered in the ER without any successful resolution of the hypotension. Physical examination by the ER doctor revealed bilateral inguinal swelling. The patient was then transferred to the MICU for blood pressure management with a Levophed® gtt. Diagnosis: After the patient's hemodynamics were stabilized a CT scan was ordered to evaluate the inguinal swelling. The CT scan revealed retroperitoneal and inguinal lymphadenopathy that was felt to resemble xxx. The patient also underwent a biopsy of the right groin, and was diagnosed as . At this time the medical team consulted oncology and a PET scan was ordered to get more conclusive information regarding the The oncology team diagnosed the patient with stage III after the PET scan showed abnormalities in the retroperitoneal, pelvic, spleen and inguinal nodes.

Polycystic Kidney Disease

Patient presentation: Thirty year old Caucasian male presents to his general practitioner for increasing back pain, which he states has become increasingly worse over the past several months. He describes the pain as "sharp and crampy," and indicates to his mid back as the location of the pain. Pt denies fever, chills, frequent urination, pain on urination or recent trauma. Past medical history is significant only for moderate hypertension, which is well controlled on current medication regimen. Family history is unobtainable. Routine labs were drawn, including CBC, CMP and urinalysis. Upon physical examination pt is found to have CVA tenderness. Deep palpation of the abdomen elicits pain bilaterally and evidences enlarged kidneys. Based on physical findings pt is sent for a CT scan. Differential list: Pyelonephritis, Nephrolithiasis, , and Renal Carcinoma Diagnosis: Significant lab results: Urinalysis revealed hematuria and proteinuria; BUN= 40 (normal 7-18) Creatinine 2.7 (normal 0.6-1.2); CT scan revealed diffuse renal cysts bilaterally; genetic testing indicates Autosomal Dominant PKD, due to mutations on either gene PKD-1 or PKD-2. XXX is a genetic disorder characterized by multiple cyst development on the kidneys. It accounts for 4% of kidney disease worldwide. There are two types: Autosomal Dominant PKD and Autosomal Recessive PKD. The recessive version is significantly less common. ADPKD is a late-onset disorder involving multiple cyst development and enlarged kidneys bilaterally. It arises from mutations on PKD-1, located on chromosome 16, or PKD-2, located on chromosome 4. These genes regulate the growth of tubular epithelium and defects in the gene cause cysts to form, resulting in damage to the kidney, loss of functional nephrons, and eventual ESRD. The XXXXX occur slowly and symptoms generally do not begin until early adulthood. The most common symptoms include back or flank pain, gross hematuria and renal stones. Individuals can have infections occur within the cysts. On initial presentation affected individuals will often have hypertension and decreasing renal function. Diagnosis is usually aided with a family history and CT, or renal ultrasound, which reveal the cystic lesions. Diagnosed patients must be followed by a nephrologist for common complications including pyelonephritis, nephrolithiasis and decreased renal function.

Hodgkin's Lymphoma

Presentation: A 26 year old female presents to her primary care doctor with complaints of weight loss, feeling of "tightness" in her chest, night sweats, fatigue, and persistent, non-productive cough times one month. She has a low grade fever of 99.5, but all other vital signs are within normal limits. Her oxygen saturation is normal, as is her breathing pattern, despite her complaints of chest heaviness. Her medical history is notable only for a tonsillectomy at age 7. Differential Diagnoses: Solid tumor cancer (likely lung), lymphoma, lung infection such as pneumonia, bronchitis, anxiety disorder with physical manifestations Diagnosis: A physical assessment is performed and the patient is noted to have swollen axillary lymph nodes. CBC with differential, CMP, ESR, and blood cultures are ordered. An EKG is obtained, and a 2 view chest x-ray is ordered. The EKG shows normal sinus rhythm. The lab work shows an elevated WBC, particularly lymphocytes, and an elevated ESR. The blood cultures are negative, ruling out infectious process. The chest x-ray shows a large, 12 cm mediastinal mass. The doctor orders a biopsy of the mass. The tissue pathology shows a presence of Reed-Sternberg cells, The doctor then orders a PET scan to detect the presence of spread of the disease. There is found to be spread to several other lymph nodes in the body.

Pancreatic adenocarcinoma

Presentation: P.S., a 59 year-old female, had a routine colonoscopy at an outside hospital, which showed 2 polyps and a mass in her ascending colon. A right colectomy for the mass was performed a few weeks later which was found to be benign per patient. However, during her admission for the colectomy, the patient developed severe abdominal pain. An MRI of the abdomen showed a 3cm mass for which the patient says she was told to follow up on as an outpatient. Two months later she became jaundiced and was readmitted to the hospital for a biliary stent and further management. Diagnosis: An outpatient CT of the abdomen/pelvis showed a 6.5 x 3.7 cm mass involving the head and body of the pancreas, which encased and narrowed the superior mesenteric artery. The patient then presented to the ER with constipation and abdominal pain. She was diagnosed with locally advanced,

Friedreich's Ataxia

Presentation: The patient, J.C., is a 19 year old male. After his wisdom teeth removal, his oral surgeon noticed that he "seemed off balance" and recommended a neurological evaluation. J.C. was physically active in high school, but reports a slight deterioration in coordination over the past months. He appears well-nourished and in good health. J.C. has no past medical history and no family history of genetic conditions exists. J.C.'s physical exam reveals a gait disturbance and mildly diminished deep tendon reflexes in his bilateral lower extremities. No evidence of scoliosis. J.C.'s speech is unaffected. Differential Diagnosis: vitamin E deficiency ataxia, some other form of ataxia, hereditary motor and sensory neuropathy type I and II Diagnosis: MRI of brain and spinal cord with IV contrast: WNL - No signs of cerebellar atrophy are present. 2D Echocardiogram: WNL - No evidence of hypertrophic cardiomyopathy present. Serum Vitamin E level: WNL A complete XX evaluation revealed a frataxin mutation on Chromosome 9. J.C. had an expanded GAA repeat of the frataxin gene, with number of repeats = 1109 and 525. XX is an autosomal recessive disorder caused by an unstable GAA trinucleotide repeat expansion (greater than 120 repeats) in the first intron of the frataxin gene on chromosome 9 in both alleles.

Invasive Pulmonary Aspergillosis

Pt B.D., a 52 year old male presents to the transplant clinic with a fever, cough, generalized weakness and, weight loss. The patient is admitted to the hospital under the diagnosis of fever with an unknown origin. The relevant medical history of the patient consists of a small bowel transplant 4 months prior. Two months after the transplant the patient was treated for rejection. The rejection treatment consisted of bolus steroids. This treatment was unsuccessful and a second treatment regimen was initiated. Twelve doses of Thymoglobulin were given over twelve days followed by improvement in the grafted small bowel and a decrease in the level of organ rejection. That patient was sent home on additional immunosuppressant medications as follows, tacrolimus, prednisone and myfortic acid. Prophylaxis treatment against viral, fungal, and bacterial infections was continued throughout the rejection treatment. Prophylaxis treatment was also continued upon discharge and the patient was sent home on Valcyte, Nystatin, and Bactrim. Possible causes of the patient's symptoms could have been as follows, bacterial infection, viral infection, or fungal infection. A possible source of the infection could have been the patients PICC line which was removed upon patients admission to the hospital, abdominal drain or gastrointestinal tube. All of the patient's invasive lines, blood, and sputum were cultured to find the source of infection. The patient was prophylactically started on Zosyn, Diflucan, and ganciclovir. Fevers were relieved with Tylenol. A chest x-ray was obtained where multiple nodules were discovered. Infectious disease and pulmonologists were consulted. An additional chest CT was done to closer examine the nodules. Upon the results of the chest CT pulmonologists recommend biopsies of the nodules done. Both sputum culture and nodule biopsies confirmed the diagnosis of Aspergillosis a fungal invasive infection in both lungs. Due to the patient's medical history the patient was a prime candidate for a fungal infection of this nature. The patient was on long term immunosuppressants and steroids. He had also recently received high doses of immunosuppressants to fight the rejection of his transplanted organ. He also had a poor nutritional state that also inhibited his body's defense system due to his transplant. As a result he was not able to fight off this fungal infection.

Syndrome of Inappropriate Antidiuretic Hormone (SIADH)

R.B., a 68 year old female, is admitted to the ED with a one month history of increased confusion and agitation. She has a history of hypertension and hypercholesterolemia. CT of the head reveals a large enhancing mass in the occipital region. MRI of the head reveals a large butterfly lesion at the splenium of the corpus callosum. On assessment patient is confused and mumbling words, unable to follow commands, is irritated and agitated. Pupils are sluggish and reactive. Pt scores 8 on the Glasgow Coma scale. Patient was intubated for increasing lethargy and unresponsiveness. Repeat CT scan of the head of the head shows increasing edema, and increased intraventricular hemorrhage with developing hydrocephalus. Two ventricular drains are placed for monitoring of ICP and drainage of blood in the ventricles. ICP initially is 9-11 (normal 7 - 15 mmHg). Two days after admission R.B.'s urine output is decreased at 15-20 cc/hr and is amber and concentrated. ICP 15-17. BUN and creatinine are within normal limits - creatinine 0.9 and BUN 11. CVP reading taken and is 10 mmHg. Mucous membranes are moist and show no signs of dehydration. Patient is in normal sinus rhythm, with her heart rate at baseline in low 70s. Blood pressure is 120s/70s. Patient is started on Decadron® IV to decrease cerebral swelling. Differential Diagnosis: Cerebral Salt Wasting Diagnosis: It is suspected that R.B. has SIADH. Labs are ordered (BMP, serum osmolarity, urine sodium, and urine osmolarity). Serum Na+ 130 mEq/L, Serum osmolarity 268 mOsm/L, urine sodium 23, Specific gravity > 1.020, urine osmolarity >200 mOsm/L. Because of the above lab values and pt is not hypovolemic she is diagnosed with S

Graft vs Host Disease (GVHD)

S.K., a 26 yr old female diagnosed with AML, presents with diarrhea; 8-12 watery movements per day, intractable vomiting that is not responsive to anti-emetics, and severe abdominal pain rating a 10/10. S.K. is 22 days s/p an allogeneic bone marrow transplant from an unknown unrelated donor. Patient developed symptoms acutely and at the time of presentation was still hospitalized awaiting full engraftment of donor marrow. Pt relevant labs at time: WBC 1.0, ANC 0.4, Hgb 6.8, Hct 22%, PLTs 12, and positive fecal occult blood test. Differential List: Inflammatory Bowel Disease, Infectious colitis (viral) - cytomegalovirus, Infectious colitis- (bacterial) Clostridium difficile, Diagnosis: An upper and lower endoscopy was performed in which findings showed mucosal edema and diffuse bleeding throughout the entire GI tract. Histological findings from biopsies of GI tract revealed crypt cell necrosis. S.K. was diagnosed with Grade IV of the GI tract, which is characterized by more than 2L of diarrhea per day positive for blood in the stool.

Bacterial Vaginosis

SD, an 18 year old teenager came to the labor and delivery unit complaining of preterm contractions that had started three days prior to arrival. She was 26 weeks pregnant, and it was her first pregnancy. She had a history of genital herpes and asthma. She was taking albuterol as needed for her asthma. Her vital signs were HR 87; BP 126/72; T 37.2 degrees Celsius; SaO2 99% on room air. When the patient was placed on the external fetal monitor, the tocometer (contraction monitor) showed that the patient was contracting every 2 to 3 minutes. When the resident put the patient in lithotomy position to perform a speculum exam to see if her cervix was dilated, a fishy smell permeated the room. The resident noticed homogeneous, thin, grayish-white discharge that smoothly coated the vaginal walls. When questioned, the patient stated that she had been having an increase in thin, off-white discharge for the past three months. She had assumed it was a normal sign of pregnancy. The patient did not feel any of the contractions that were showing up on the external fetal monitor. Differential Diagnosis: Trichomoniasis Yeast Infection Preterm Labor Diagnosis: The resident took a sample of the discharge and placed it on the glass side to examine it under the microscope. Clue cells were noted on the saline wet mount. The patient's cervix was long and closed which ruled out preterm labor. The patient denied vaginal itching, and the discharge did not look like yeast. A positive whiff test helped confirm the diagnosis

Wegener's Granulomatosis

atient Presentation KMM was a 58-year-old female who presented to the ER complaining of frequent headaches and a cough for approximately 2 months. Additional symptoms on arrival to the ER included nosebleeds, worsening shortness of breath, generalized weakness, and recent weight loss. Past medical history included hyperlipidemia, osteoarthritis, thyroid disorder, fatty liver disease, and diabetes. Initial vital signs: BP 132/76, HR 79, RR 17, T 36.8, Oxygen saturation 95% on room air. Differential Diagnosis Pneumonia, influenza, Lyme disease, lupus, glomerulonephritis Diagnosis Labs were drawn on KMM to rule out infection, including blood cultures x2 and a UA/UC to rule out a UTI. All cultures showed no growth to date. The urinalysis showed significant red blood cells present. Anti-neutrophil cytoplasmic antibodies (ANCA) were positive in the blood, which are consistent but not diagnostic of XXX KMM had an abnormal chest x-ray, which was followed up with a CT scan. The scan showed both cavitating and non-cavitating nodules. Based on the abnormal CT, physicians ordered a bronchoscopy with biopsy. The lung biopsy was diagnostic of XXX, an autoimmune disorder that causes blood vessels to become inflamed.

Bacterial vaginosis/vaginitis

atient Presentation The patient, B.B., a 21 year old, G1P0 at 36 weeks gestation presented to the clinic for her scheduled prenatal visit. Her pregnancy had been uncomplicated and progressing along well. When the patient was asked if she had any problems or concerns, she reported increased vaginal discharge, vaginal itching and odor for about one week. The patient denied dysuria or dyspareunia. When asked why she had not reported these symptoms sooner, she explained that she thought the discharge was normal at first and expected the symptoms to resolve. Differential Diagnosis , chlamydia, gonorrhea, trichomoniasis Diagnosis The patient was prepared for a speculum exam, and the exam revealed increased milky white vaginal discharge and a "fishy" odor. A sample of the discharge was taken and a wet mount was created which showed clue cells and a positive whiff test. The vaginal discharge was tested with nitrazine and found to have a pH around 5. Other then the findings from the speculum exam, the patient's physical portion of the visit was benign (i.e. no CVA tenderness or abdominal pain). The patient's symptoms and findings from the wet mount led to a diagnosis ofXXXX

Breast Cancer

atient Presentation: J.D., a 44 year old woman presents at the office after noting a non-tender nodule, about the size of a marble, on her right breast. Pt is G3P2 with her first pregnancy at age 28. Pt currently is not breastfeeding. Last menses was 1 week ago and normal. Patient tracked the nodule through the last menses to see if it resolved but it did not. Pt does regular SBE each month and noticed the lump in this manner. No h/o breast problems or symptoms. Patient's maternal grandmother was diagnosed with breast cancer at age 75. No other family h/o breast cancer. No recent trauma or h/o breast implants. Differential Diagnoses Breast abscess, fibrocystic breasts, carcinoma Diagnosis A careful breast examination was performed to rule out an infection (the mass would be tender), which it was not. The mass also did not transilluminate which can be indicative of a cyst. The lymph nodes in the axilla and neck were not enlarged. The mass was located in the UOQ of her right breast and was asymmetrical, meaning it was only found on one breast. It was firm, mobile, and located around 11 o'clock. The provider sent the patient to a Comprehensive Breast Center for a diagnostic mammogram and right breast ultrasound. The mammogram showed dense breast tissue with possible architectural distortion but no malignant calcifications in the lump. The ultrasound defined the lump as an irregular mass that was 0.8 x 0.7 x 1.9 cm. The radiologist also performed an u/s guided needle biopsy of the lesion.


Kaugnay na mga set ng pag-aaral

Chapter 34: Child Health Assessment

View Set

Sociology First Test Sample test

View Set

A&P Ch 4 Skin and Body Membranes

View Set

2 - Life/Health Insurance Underwriting

View Set

ACCT Chapter 11 Property, Plant, and Equipment and Intangible Assets: Utilization and Disposition

View Set